Схема дизельного двигателя: Ошибка 404 — страница не найдена

Содержание

Принцип работы дизельного двигателя. — Автомастер

Принцип работы дизельного двигателя.

Подробности

Принцип работы дизельного двигателя немного отличается от принципа работы бензинового. Отличие это состоит в том, что смесеобразование происходит уже внутри самого цилиндра, у бензинового же двигателя приготовление смеси происходит снаружи. В цилиндр она подается уже готовой. Существенным отличием является воспламенение рабочей смеси. В бензиновом двигателе воспламенение происходит от свечи зажигания, а в дизельном происходит самовоспламенение.

    Теперь разберем рабочие циклы четырехтактного дизельного двигателя:
  1. Такт впуска.

    Рис 1 – Такт впуска.

    1 – впускной клапан. 2 – выпускной клапан. 3 – топливная форсунка.

    За первый такт, поршень перемещается от верхней мертвой точки ВМТ к нижней НМТ. Впускной клапан 1 открыт, выпускной 2 закрыт. За счет создаваемого разрежения в цилиндре, вовнутрь устремляется порция воздуха.
  2. Такт сжатия.

    Рис 2 — Такт сжатия.

    На этом этапе, оба клапана как впускной, так и выпускной закрыты. Поршень перемещается из НМТ в ВМТ, сжимая воздух. Давление в камере достигает 5 МПа, а температура воздуха за счет сжатия возрастает до 700 градусов Цельсия.
  3. Такт расширения или рабочий ход.

    Рис 3 — Такт расширение. Рабочий ход.

    При достижении поршнем верхней мертвой точки (при максимальном давлении в цилиндре), через форсунку, под высоким давлением, создаваемым топливным насосом закачивается порция топлива. Форсунка распыляет топливо, которое смешиваясь с горячим воздухом самовоспламеняется. В результате горения, температура в камере резко повышается до 1800 градусов Цельсия, вместе с ней в разы увеличивается и давление 11 МПа. Поршень, передвигаясь от верхней мертвой точки к нижней мертвой точки, совершает полезную работу. В конце такта температура падает до 700 — 800 градусов, давление снижается до 0.
    3 – 0.5 МПа.
  4. Такт выпуска.

    Рис 4 – Такт выпуска.

    Выпускной клапан 2 открывается, и поршень выталкивает отработанные газы. Температура и давление опускаются до 500 градусов и 0.1 МПа.

Далее рабочие циклы повторяются.

Подробнее об устройстве и осбеностях конструкции дизельных двигателей.

Система питания дизельного двигателя, схема устройства

Устройство системы питания дизельного двигателя

На классических дизелях система питания состоит из топливного бака, фильтров грубой и тонкой очистки топлива, топливного насоса высокого давления (ТНВД) и форсунок.

Система питания дизельного двигателя современного дизеля управляется электроникой, а набор датчиков примерно такой же, как у бензиновых моторов.

Топливо из топливного бака по трубопроводу поступает в топливный фильтр, а затем в топливный насос высокого давления (ТНВД). Для защиты элементов питания от попадания в них воды, помимо топливного фильтра в трубопровод может быть установлен водоотделитель.

Насос нагнетает топливо в форсунки.

На старых дизелях форсунки были механическими. На современных дизелях топливные форсунки электромагнитные. Работой электромагнитных форсунок, так же, как и в бензиновом двигателе, управляет электроника на основании сигналов, поступающих от датчиков системы. Что бы ни случилось, в любой ситуации наши специалисты по выездной тех помощи на дорогах москвы приедут и окажут необходимую помощь.

Излишки топлива от форсунок поступают в обратную магистраль. Из следующей главы можно будет узнать система выпуска отработавших газов, описание и схема.

Система питания дизельного двигателя с турбонаддувом

Для повышения мощности в современных

системах питания дизельного двигателя широко используется турбо-наддув, который позволяет увеличить количество поступающего в цилиндры воздуха. В результате возрастает крутящий момент двигателя. А в одной из следующих глав можно будет узнать неисправности двигателя, неисправности систем двигателя: список из перечня неисправностей и условий, при которых запрещается эксплуатация транспортных средств. .

Как работает система питания дизельного двигателя с турбонаддувом? Очищенный фильтром воздух по воздуховоду проходит к турбонагнетателю.

В воздуховоде установлен датчик массового расхода, который сообщает информацию о количестве проходящего воздуха в электронный блок управления дизелем.

Турбонагнетатель установлен на выпускной трубопровод и приводится в действие энергией отработавших газов. Из турбонагнетателя воздух проходит к впускному трубопроводу.

Для снижения температуры воздуха применяется интеркулер (промежуточный охладитель).

После интеркулера воздух подводится через впускной трубопровод к впускным клапанам цилиндров.

Для облегчения запуска холодного дизельного двигателя применяются специальные свечи подогрева, установленные в камеры сгорания.

Свечи включаются по команде электронного блока управления после поворота ключа в замке и работают в течение нескольких секунд.

После выключения свечей на щитке приборов гаснет контрольная лампа, и двигатель можно запускать.

Принцип действия дизельных двигателей. Индикаторные и круговые диаграммы

Дизелем называют ДВС с внутренним смесеобразованием, в котором тяжелое жидкое топливо, вводимое в распыленном состоянии в цилиндр в конце хода сжатия, самостоятельно воспламеняется в горячем сжатом воздухе. Основными понятиями, относящимися ко всем дизельным двигателям, являются (рис. 17):

  • верхняя мертвая точка (ВМТ) – положение поршня, при котором он наиболее удален от оси коленчатого вала;
  • нижняя мертвая точка (НМТ) – положение поршня наиболее близкое к оси коленчатого вала;
  • ход поршня S , [м] – расстояние между ВМТ и НМТ: S = 2R ;
  • рабочий объем цилиндра VS , [м3] – объем, описываемый поршнем при движении между ВМТ и НМТ :
  • объем камеры сжатия VC , [м3] – объем цилиндра над поршнем при нахождении его в ВМТ;
  • полный объем цилиндра VA , [м3] – сумма рабочего объема цилиндра и объема камеры сжатия:

Принцип действия четырехтактного дизеля

Рабочий цикл в цилиндре четырехтактного дизеля осуществляется за два оборота коленчатого вала (4 хода поршня).

Цилиндр четырехтактного дизеля закрыт крышкой, в которой располагаются клапаны для впуска свежего заряда воздуха и выпуска продуктов сгорания (рис. 18). Впускные и выпускные клапаны удерживаются в закрытом положении пружинами и давлением, создаваемым в цилиндре в периоды сжатия, сгорания топлива и расширения. Открытие клапанов в необходимые моменты времени осуществляется с помощью газораспределительного механизма.

Рабочий цикл четырехтактного дизеля состоит из следующих процессов (тактов): впуска, сжатия, расширения (рабочего хода) и выпуска, и происходит следующим образом (рис. 18):

Первый такт – впуск. В начальный момент времени давление в цилиндре двигателя несколько выше атмосферного – точка 1 индикаторной диаграммы (рис. 18). Поршень из ВМТ начинает свое движение к НМТ, открывается впускной клапан и поршень всасывает в цилиндр свежий заряд воздуха (процесс 1− 2). При этом давление в цилиндре устанавливается чуть ниже атмосферного (для двигателей без наддува) за счет гидравлического сопротивления впускного клапана.

Часто для увеличения массы свежего заряда воздух предварительно сжимают в компрессоре до избыточного давления 0,13 ÷ 0,4 МПа, а затем охлаждают в воздухоохладителе. Такое увеличение массы свежего заряда называется наддувом.

Второй такт – сжатие. Поршень из НМТ начинает движение к ВМТ. Впускной клапан закрывается и происходит сжатие воздуха, поступившего в цилиндр дизеля. При этом уменьшается объем заряда воздуха, повышается его давление (процесс 2 − 3 ) до 3,6 ÷ 4,0 МПа в дизелях без наддува, а при высоком наддуве – до 11,0 МПа, что сопровождается увеличением температуры воздуха до 500 °C и выше. В конце такта, при нахождении поршня вблизи ВМТ, в цилиндр через форсунку начинает поступать мелко распыленное топливо, которое от соприкосновения с горячим воздухом самовоспламеняется и начинает гореть. При сгорании топлива давление в цилиндре повышается до 5,5 ÷ 8,5 МПа в дизелях без наддува, и до 11,0 ÷ 14,5 МПа в дизелях с высокой степенью наддува. Процесс сгорания ~ 40 % топлива в конце такта сжатия близок к изохорному (изображен на индикаторной диаграмме линией 3 − 4 ) и происходит при нахождении поршня вблизи ВМТ.

Третий такт – расширение (рабочий ход). В начале такта расширения топливо продолжает поступать в цилиндр дизельного двигателя, и процесс сгорания ~ 60 % топлива при начале движения поршня от ВМТ к НМТ близок к изобарному (процесс 4 − 5 на диаграмме). По окончании сгорания топлива происходит расширение продуктов сгорания (процесс 5 − 6 на индикаторной диаграмме). Расширяющиеся продукты сгорания воздействуют на поршень, совершая полезную работу. Давление газов в цилиндре двигателя и их температура в ходе процесса расширения понижаются.

Четвертый такт – выпуск. По окончании хода расширения открывается выпускной клапан, и поршень начинает движение от НМТ к ВМТ. При этом происходит выпуск отработавших газов через выпускной клапан (процесс 6 −1 на индикаторной диаграмме). Давление в цилиндре в процессе выпуска газов несколько выше атмосферного за счет гидравлического сопротивления выпускного клапана.

Таким образом в четырехтактном дизельном двигателе полезным является только такт расширения (рабочий ход), остальные три такта осуществляются за счет кинетической энергии вращающегося коленчатого вала с маховиком и работы других цилиндров двигателя.

Процессы газообмена в цилиндре дизельного двигателя (фазы газораспре-деления) могут быть изображены на двух окружностях, обозначающих периоды открытия впускных и выпускных клапанов в функции угла поворота коленчатого вала. Такие диаграммы называются диаграммами газораспределения или круговыми диаграммами.

В 4-хтактных дизелях на газообмен отведено 550 ÷ 570 градусов поворота коленчатого вала (ПКВ). Процесс газообмена в четырехтактных дизелях можно разбить на следующие периоды (рис. 19):

Свободный выпуск – осуществляется за счет разницы атмосферного давления и давления в цилиндре двигателя в момент открытия выпускного клапана (линия О − А диаграммы). При этом газы с большой скоростью устремляются в выпускной патрубок двигателя. Продолжительность периода свободного выпуска примерно соответствует углу предварения открытия выпускного клапана (ϕ1 = 40 ÷ 50° ПКВ). Тепловая и кинетическая энергия выпускных газов, как правило, используется для привода турбокомпрессора или работы утилизационных котлов.

Принудительный выпуск – теоретически начинается в НМТ и заканчивается в ВМТ. Это принудительное выталкивание продуктов сгорания из цилиндра телом поршня.

Продувка – в конце хода выпуска открывается впускной клапан (линия О − С , ϕ 3 = 50 ÷ 60° ПКВ до ВМТ), а выпускной остается открытым. При двух открытых одновременно клапанах происходит продувка камеры сгорания воздухом и удаление оставшихся в цилиндре газов. Кроме того, продувка снижает температуру стенок камеры сгорания, поршня и выпускных клапанов, улучшая условия работы и увеличивая срок их службы. Продолжительность продувки составляет ~ 110 ° ПКВ.

Наполнение – теоретически начинается в ВМТ, а фактически – с момента закрытия выпускного клапана (линия O − D , ϕ 4 = 50 ÷ 55° ПКВ за ВМТ) и частично протекает одновременно с продувкой. Окончание наполнения совпадает с приходом поршня в НМТ.

Дозарядка – поршень движется вверх по ходу сжатия, а впускной клапан некоторое время остается открытым до момента, соответствующего линии O − B на диаграмме (ϕ 2 = 30 ÷ 40° ПКВ после НМТ). Воздух продолжает поступать в цилиндр по инерции и несколько увеличивает плотность заряда в цилиндре.

Принцип действия двухтактного дизеля

Из рассмотрения индикаторной диаграммы четырехтактного дизельного двигателя видно, что он только половину времени, затраченного на цикл, работает как тепловой двигатель (такты сжатия и расширения). Остальное время (такты впуска и выпуска) двигатель работает как воздушный насос. Более полно время, отводимое на рабочий цикл, используется в двухтактных дизелях, в которых рабочий цикл осуществляется за один оборот коленчатого вала. Необходимая замена отработавших газов свежим воздухом происходит на небольшой части хода поршня в конце такта расширения и в начале такта сжатия, и составляет примерно 140 ÷ 150 ° ПКВ.

В отличие от четырехтактного, в двухтактном дизеле вместо впускных и выпускных клапанов в стенке цилиндра выполнены впускные (продувочные) ПО и выпускные ВО окна (рис. 20). Продувочным насосом ПН воздух нагнетается в воздушный ресивер Р, и через продувочные окна ПО поступает в цилиндр двигателя. Продукты сгорания топлива покидают цилиндр через выпускные окна ВО и выпускной патрубок ВП. Открытие и закрытие продувочных и выпускных окон осуществляется телом поршня при его движении в цилиндре.

Рабочий цикл двухтактного дизеля изображен на рис. 21 и состоит из следующих тактов:

Первый такт – сжатие. Поршень находится в НМТ. Продувочные и выпускные окна полностью открыты. При этом происходит продувка цилиндра, продолжающаяся до тех пор, пока поршень, двигаясь вверх, не перекроет продувочные окна (процесс 7 − 6 на диаграмме). При последующем движении поршень закроет выпускные окна, причем в период, изображенный на диаграмме линией 6 −1, из цилиндра выталкивается часть свежего заряда воздуха. После закрытия поршнем выпускных окон, начинается сжатие воздуха, сопровождающееся повышением давления и температуры (процесс сжатия изображен на диаграмме линией 1− 2 ). При подходе поршня к ВМТ в цилиндр впрыскивается мелко распыленное топливо, которое воспламеняется от соприкосновения с горячим воздухом. Часть топлива (~ 40 %) сгорает при постоянном объеме при нахождении поршня вблизи ВМТ (процесс 2 − 3).

Второй такт – рабочий ход (расширение). Поршень начинает движение от ВМТ к НМТ. Оставшаяся часть топлива (~ 60 %) сгорает при постоянном давлении (процесс 3 − 4 ). После полного сгорания топлива происходит расширение горячих газов (линия 4 − 5 ), которое заканчивается, когда поршень своей кромкой откроет выпускные окна в точке 5. С этого момента начинается свободный выпуск отработавших газов, сопровождающийся резким понижением давления в цилиндре (процесс 5 − 6 ). В точке 6 поршень открывает продувочные окна и начинается продувка цилиндра – принудительное вытеснение из него потоком воздуха отработавших газов и заполнение свежим зарядом воздуха (процессы 6 − 7 и 7 − 6 на диаграмме).

Теоретически при одинаковых размерах цилиндра и равных числах оборотов в минуту двухтактный дизель может развивать мощность в 2 раза большую, чем четырехтактный. В действительности мощность двухтактного дизеля (при прочих равных условиях) больше лишь в 1,7 ÷ 1,8 раза, чем у четырехтактного, так как часть хода поршня затрачивается на процессы выпуска и продувки. Кроме того на привод навешенного на двигатель продувочного насоса затрачивается 6 – 8 % мощности двигателя.

Весь процесс газообмена двухтактного дизеля можно условно разделить на следующие периоды (рис. 22):

Свободный выпуск – начинается с момента открытия поршнем выпускных окон (линия О − b ) и заканчивается в момент открытия поршнем продувочных окон (линия O − d ). В этот период происходит интенсивный выброс отработавших газов в выпускной тракт за счет перепада давлений в цилиндре (~ 0,45 МПа) и в выхлопном патрубке (~ 0,14 МПа).

Принудительный выпуск и продувка – начинаются в точке d и заканчиваются в момент закрытия продувочных окон (линия O − d ′ ). При этом происходит принудительное вытеснение отработавших газов продувочным воздухом и одновременное заполнение цилиндра свежим зарядом.

Потеря заряда воздуха – объясняется тем, что верхние кромки выпускных окон расположены выше продувочных. Поршень при движении к ВМТ до момента закрытия выпускных окон (линия O − a ) успевает вытолкнуть через выпускные окна часть поступившего в цилиндр воздуха. Фаза потери заряда воздуха является нежелательной, поэтому существует ряд конструктивных решений для замены ее на фазу дозарядки. Например, вместо щелевой схемы продувки, описанной выше, используют прямоточную клапанно-щелевую схему. В таких конструкциях дизелей выпускные окна отсутствуют, а вместо них в крышке цилиндра устанавливается выпускной клапан, приводимый в действие от механизма газораспределения.

Литература

Судовые энергетические установки. Дизельные и газотурбинные установки. Болдырев О.Н. [2003]

Похожие статьи

Для семейства моделей KIA Ceed предложена силовая установка по схеме «мягкого гибрида»

9 июля 2020 г.

Франкфурт-на-Майне, 9 июля 2020 – KIA Motors начинает предлагать для всех моделей, входящих в популярное семейство Ceed, силовые установки EcoDynamics+, выполненные по схеме «мягкого гибрида» (MHEV) с рабочим напряжением 48В на базе дизельного двигателя. Высокоэффективный новый силовой агрегат становится доступен для пятидверного хэтчбека Ceed, универсала Ceed Sportswagon, пятидверного «шутинг брейка» ProCeed и городского кроссовера XCeed.

Силовые установки EcoDynamics+ обеспечивают повышение эффективности предлагающегося для семейства Ceed дизельного двигателя линейки Smartstream с рабочим обемом 1,6 л, позволяют снизить уровень выбросов CO2 и добиться снижения затрат на эксплуатацию автомобиля. Новые силовые агрегаты будут доступны на базе обоих вариантов дизельных двигателей – базового (115 л.с.), и увеличенной мощности (136 л.с.). Для ProCeed предусмотрена только более мощная версия установки MHEV.

Внедрение новой «мягко-гибридной» технологии для всей линейки моделей Ceed позволяет добиться приблизительно 5-10-процентного снижения выбросов CO2 (замеры по комбинированному циклу NEDC 2.0), в зависимости от конкретного типа кузова, мощности конкретного двигателя и комплектации автомобиля. Принципиально важным моментом является то, что внедрение технологии MHEV позволяет повысить топливную эффективность работы силового агрегата без необходимости идти на компромисс в плане динамики автомобиля.

«Мягко-гибридные» версии моделей KIA Ceed, Ceed Sportswagon, ProCeed и XCeed с силовыми установками EcoDynamics+ на базе дизельных двигателей – выпускаются на европейском заводе бренда, в словацком городе Жилина. Автомобили уже доступны для приобретения и заказа во многих странах Европы, на новые версии распространяется такаяже уникальная по своей длительности гарантия производителя, как и на остальные модели современного модельного ряда KIA.

Силовая установка MHEV EcoDynamics+ на базе дизельного двигателя имеет режимы «мотор» и «генератор»

Базовым для мягко-гибридной силовой установки стал самый «чистый» дизельный двигатель, когда-либо производившийся под маркой KIA – 1,6-литровый агрегат линейки Smartstream. Силовая установка MHEV EcoDynamics+ с рабочим напряжением 48В позволяет повысить его топливную эффективность за счет направления в необходимые моменты вспомогательного потока крутящего момента.

Силовые установки EcoDynamics+ разработаны, чтобы добиться более высокой топливной эффективности двигателей внутреннего сгорания в условиях реальной эксплуатации автомобиля. В основе принципа их действия – незаметная рекуперация электрической энергии и отдача ее в необходимые моменты. В таких ситуациях мягко-гибридная система оказывает двигателю поддержку дополнительным потоком крутящего момента за счет накопления запаса энергии в компактной батарее литий-ионных полимерных аккумуляторов с рабочим напряжением 48 Вольт. Наличие в системе стартер-генератора (MHSG – Mild Hybrid Starter-Generator) позволяет двигателю больше времени находиться в заглушенном состоянии.

Стартер-генератор MHSG соединен ременным приводом с коленчатым валом двигателя, он способен незаметно для водителя переключаться между режимами «мотор» и «генератор». В режиме «мотор», при разгоне автомобиля, MHSG обеспечивает «электрическую» помощь, что позволяет снизить нагрузку на основной двигатель и снизить уровень вредных выбросов. Когда автомобиль замедляется, MHSG при соблюдении определенных условий – незаметно переключается в режим генератора, обеспечивая рекуперацию энергии от вращения коленчатого вала и подзарядку аккумуляторной батареи.

В 1,6-литровом дизельном двигателе KIA U3 CRDi (с общей топливной рампой) используется технология селективной каталитической нейтрализации SCR – эта технология активного управления выхлопными газами позволяет добиться существенного снижения вредных выбросов. Таким образом, двигатель выбрасывает в воздух меньше углекислого газа, твердых частиц и оксидов азота (NOx), чем какие-либо более ранние дизельные двигатели KIA. Новые двигатели предлагаются в двух вариантах мощности – с прошивками на 115 л.с. или на 136 л.с.

«Интеллектуальная» механическая трансмиссия iMT с электронным управлением сцеплением Clutch-by-wire обеспечивает дополнительное повышение топливной эффективности.

Ceed также стал одной из первых моделей KIA, которая доступна в Европе с новейшей разработкой бренда – «интеллектуальной» механической трансмиссией iMT. Она предлагается исключительно в паре с «мягко-гибридными» силовыми установками EcoDynamics+. Система электронного управления сцеплением дополняет возможности MHEV в плане более высокой топливной эффективности и снижения выбросов CO2 . Ключевым моментом является то, что при этом автомобиль сохраняет столь же интересный для водителя характер динамики, что предлагают модели с традиционной механической трансмиссией.

В трансмиссии iMT управление приводом сцепления осуществляется без механической связи с педалью – исключительно при помощи электронных сигналов. Такая система беспрепятственно сочетается с силовой установкой EcoDynamics+. Трансмиссия iMT в паре с MHSG позволяет при движении автомобиля накатом с замедлением – отключать основной двигатель раньше, чем это делает система временного выключения двигателя KIA Stop & Go. В «экологичном» режиме Eco, который устанавливается у Ceed по умолчанию – короткие отключения двигателя при движении накатом возможны при скоростях до 125 км/ч, при этом как только водитель начинает нажимать педаль газа или сцепления – происходит незаметный перезапуск двигателя. iMT в условиях реальной эксплуатации обеспечивает дополнительное повышение топливной эффективности мягко-гибридной силовой установки, а также дополнительное снижение выбросов СО2 приблизительно на 3%.

Выбранная водителем передача остается задействованной в то время, когда происходит временная приостановка работы двигателя. Двигатель перезапускается на этой же передаче, как только водитель нажимает педаль газа или тормоза – перезапуск происходит за счет потока энергии от стартер-генератора MHSG. Однако, если водитель начинает действия с нажатия педали сцепления (то есть намерен переключить передачу), или если скорость автомобиля упала ниже оптимальной для ранее выбранной передачи – система перезапускает двигатель в «нейтральном» положении iMT, с разомкнутым сцеплением.

Снижение выбросов CO2 , более низкие затраты водителей на топливо

Новая мягко-гибридная силовая установка KIA EcoDynamics+ на базе дизельного двигателя – вносит свой вклад в повышение топливной эффективности и снижение эксплуатационных расходов моделей Ceed, Ceed Sportswagon, ProCeed и XCeed.

Показатель выбросов CO2 для оснащенных силовыми установками EcoDynamics+ моделей Ceed и Ceed Sportswagon находится на низком уровне: всего 96 г/км при агрегатировании с базовой трансмиссией iMT, и 99 г/км при работе в паре с предлагающейся в качестве опции семиступенчатой роботизированной преселективной трансмиссией с двумя сеплениями 7DCT (замеры в комбинированном цикле NEDC 2.0). Таким образом, улучшение показателей составляет, соответственно, 5,0% или 9,2% по сравнению с таким же 1,6-литровым дизельным двигателем Smartstream без использования 48-вольтовой мягко-гибридной технологии.

Для модели ProCeed показатели выбросов в комбинированном цикле составляют всего 103 г/км для версий с трансмиссией iMT, и 104 г/км для автомобилей с трансмиссией 7DCT. В данном случае улучшение показателей составляет 7.2% или 6,3% соответственно.

Аналогично, для городского кроссовера XCeed выбросы находятся на уровне 101 г/км (iMT) или 108 г/км (7DCT) – улучшение составляет 7,3% или 6,1%.*

ЭЛЕКТРИЧЕСКАЯ схема управления дизельного двигателя toyota prado 120 ПОМОГИТЕ ПОЖАЛУЙСТА ​

Ответ:

В большинстве регионов нашей страны, особенно за Уралом, под словом внедорожник как правило подразумевается одна конкретная и единственная модель — Toyota Land Cruiser. Уровень почитания этого «джипа» таков, что клуб владельцев напоминает закрытую секту, где запрещено говорить о машине в отрицательном контексте. Но возраст и законы физики никто из автопроизводителей отменить не в силах, так что очевидно, что и у «Крузаков» есть немало проблем. В общем, разбираемся с подержанным Prado третьего поколения.Перед тем как разбираться с техническими нюансами Toyota Land Cruiser, об этой машине необходимо знать немало важных вещей, которые могут отбить желание ее покупать, даже не подходя к просмотру объявлений.

Во-первых, речь идет о немалой цене подержанных экземпляров. Легенда о неубиваемости Toyota в нашей стране породила сильнейший ценовой перекос: 12-15 летние экземпляры Prado с чудовищными пробегами по стоимости сегодня могут превосходить новые кроссоверы RAV4.

Фото: Пресс-служба АвтоВАЗа

Первая Lada с вариатором. На что способен новый Xray Cross

Соответственно, всегда считавшиеся «крутыми» и «авторитетными» «Крузаки» пользовались популярностью и в криминальном мире. Модель всегда числилась в первых строчках рейтингов по угонам, а на вторичном рынке встречается немало экземпляров с темным прошлым.

Очень комфортный и проходимый внедорожник нередко становился в богатой семье автомобилем для путешествий. Годовой пробег 35-50 тыс.км для Land Cruiser не редкость даже в регионах, так что к сегодняшнему дню средний километраж Prado 120 превосходит 300 000 км, что для большинства узлов машины, включая отдельные двигатели, является границей ресурса. Беда в том, что даже такой пробег в большинстве случаев является «скрученным», и сколько машина пробежала в реальности можно только догадываться.

Особая категория Land Cruiser — экземпляры, пригнанные из-за рубежа. Основных регионов привоза было три — США, Ближний Восток и Япония. Все три, как правило, проблемные. Из США машины попадали со всевозможных аукционов, где «за копейки» выкупались поврежденные в ДТП и стихийных бедствиях экземпляры — так называемые «утопленники» и «битки».

Фото: www.facebook.com/Kurdistanautomotive

Новый Toyota Land Cruiser 200 позволил рассмотреть детали

Дешевые экземпляры с Востока имели свои особенности, в частности, отсутствие антикоррозийной обработки, из-за чего сгнивал номер рамы, а также диковинные версии без печки, но с двумя кондиционерами. Ну а «праворукие» версии из Японии могут оказаться жертвами «распила».

В общем, имейте ввиду, покупать Land Cruiser Prado с рук нужно с отрядом автомобильных экспертов. Какую-то гарантию могут дать разве что площадки trade-in официальных дилеров, которые обычно не связываются с проблемными машинами.

Объяснение:

Дизельный двигатель типы дизельного двигателя и принцип его работы

Топливо в дизельных двигателях воспламеняется от соприкосновения со сжатым воздухом.

Бензиновый двигатель является довольно неэффективным и способен преобразовывать всего лишь около 20-30% энергии топлива в полезную механическую работу. Стандартный дизельный двигатель обычно имеет коэффициент полезного действия в 30-40%, а с турбонаддувом и промежуточным охлаждением свыше 50% (например, MAN S80ME-C7 тратит только 155 гр на кВт*ч, достигая эффективности 54,4%). Дизельный двигатель из-за использования впрыска высокого давления не предъявляет требований к летучести топлива, что позволяет использовать в нём низкосортные тяжелые масла (даже на подсолнечном масле дизель может работать практически без потери мощности).
Дизельный двигатель не способен развивать высокие обороты— смесь не успевает догореть в цилиндрах, что приводит к снижению удельной мощности двигателя на 1 л объёма, а значит, и к снижению удельной мощности на 1кг массы двигателя. Это послужило причиной малого распространения дизелей в авиации (только некоторые бомбардировщики Юнкерс, а также советский тяжелый бомбардировщик Пе-8 и Ер-2, оснащавшиеся авиационными дизелями АЧ-30 и АЧ-40 конструкции А.Д.Чаромского и Т.М.Мелькумова). На максимальной эксплуатационной мощности смесь в дизеле не догорает, приводя к выбросу облаков сажи (есть народная поговорка «тепловоз дает медведя»).
Дизельный двигатель не имеет дроссельной заслонки, регулирование мощности осуществляется регулированием количества впрыскиваемого топлива. Это приводит к отсутствию снижения давления в цилиндрах на низких оборотах. Потому дизель выдаёт высокий вращающий момент при низких оборотах, что делает автомобиль с дизельным двигателем более «отзывчивым» в движении, чем такой же автомобиль с бензиновым двигателем. По этой причине в настоящее время большинство грузовых автомобилей оборудуются дизельными двигателями. Это является преимуществом также и в двигателях морских судов, так как высокий крутящий момент при низких оборотах делает более лёгким эффективное использование мощности двигателя.

По сравнению с бензиновыми двигателями, в выхлопных газах дизельного двигателя, как правило, меньше окиси углерода (СО), но теперь, в связи с применением каталитических конвертеров на бензиновых двигателях, это преимущество не так заметно. Основные токсичные газы, которые присутствуют в выхлопе в заметных количествах— это углеводороды (НС или СН) , оксиды (окислы) азота (Nox) и сажа (или её производные) в форме чёрного дыма. Они могут привести к астме и раку лёгких. Больше всего загрязняют атмосферу дизели грузовиков и автобусов, которые часто являются старыми и неотрегулированными.
Другим важным аспектом, касающимся безопасности, является то, что дизельное топливо нелетучее (то есть легко не испаряется) и, таким образом, вероятность восгорания у дизельных двигателей намного меньше, тем более, что в них не используется система зажигания, попросту говоря, у дизеля нет свечей зажигания. Вместе с высокой топливной экономичностью это стало причиной широкого применения дизелей на танках, поскольку в повседневной небоевой эксплуатации уменьшался риск возникновения пожара в моторном отделении из-за утечек топлива. Меньшая пожароопасность дизельного двигателя в боевых условиях является мифом, поскольку при пробитии брони снаряд или его осколки имеют температуру, сильно превышающую температуру вспышки паров дизельного топлива и так же способны достаточно легко поджечь вытекшее горючее. Детонация смеси паров дизельного топлива с воздухом в пробитом топливном баке по своим последствиям сравнима со взрывом боекомплекта, в частности, у танков Т-34 она приводила к разрыву сварных швов и выбиванию верхней лобовой детали бронекорпуса. С другой стороны, дизельный двигатель в танкостроении уступает карбюраторному в плане удельной мощности (мощности, снимаемой с единицы массы мотора), а потому в ряде случаев (высокая мощность при малом объёме моторного отделения) более выигрышным может быть использование именно карбюраторного силового агрегата.

Конечно, существуют и недостатки, среди которых характерный стук дизельного двигателя при его работе и маслянистость топлива. Однако, они замечаются в основном владельцами автомобилей с дизельными двигателями, а для стороннего человека практически незаметны.

Явными недостатками дизельных двигателей являются необходимость использования стартера большой мощности, помутнение и застывание летнего дизельного топлива при низких температурах, сложность в ремонте и регулировке топливной аппаратуры (ТНВД), так как насосы высокого давления являются устройствами, изготовленными с высокой точностью. Также дизель-моторы крайне чувствительны к загрязнению топлива механическими частицами и водой. Такие загрязнения очень быстро выводят топливную аппаратуру из строя. Ремонт дизель-моторов, как правило, значительно дороже ремонта бензиновых моторов аналогичного класса. Литровая мощность дизельных моторов также, как правило, уступает аналогичным показателям бензиновых моторов, хотя дизель-моторы обладают более ровным крутящим моментом в своём рабочем диапазоне. Экологические показатели дизельных двигателей значительно уступали до последнего времени двигателям бензиновым. На классических дизелях с механически управляемым впрыском возможна установка только окислительных нейтрализаторов отработавших газов («катализатор» в просторечии), работающих при температуре отработавших газов свыше 300°C, которые окисляют только CO и CH до безвредных для человека углекислого газа (CO2) и воды. Также раньше данные нейтрализаторы выходили из строя вследствие отравления их соединениями серы (количество соединений серы в отработавших газах напрямую зависит от количества серы в дизельном топливе) и отложением на поверхности катализатора частиц сажи. Ситуация начала меняться лишь в последние годы в связи с внедрением дизелей так называемой «Common-rail» системы. В данном типе дизелей впрыск топлива осуществляется электрически управляемыми форсунками. Подачу управляющего электрического импульса осуществляет электронный блок управления, получающий сигналы от набора датчиков. Датчики же отслеживают различные параметры двигателя, влияющие на длительность и момент подачи топливного импульса.

Так что, по сложности современный и экологически такой же чистый, как и бензиновый дизель-мотор ничем не уступает своему бензиновому собрату, а по ряду параметров сложности и значительно его превосходит. Так, например, если давление топлива в форсунках обычного дизеля с механическим впрыском составляет от 100 до 400 бар, то в новейших системах «Common-rail» оно находится в диапазоне от 1000 до 2500 бар, что влечёт за собой немалые проблемы. Также каталитическая система современных транспортных дизелей значительно сложнее бензиновых моторов, так как катализатор должен «уметь» работать в условиях нестабильного состава выхлопных газов, а в части случаев требуется введение так называемого «сажевого фильтра». «Сажевый фильтр» представляет собой подобную обычному каталитическому нейтрализатору структуру, устанавливаемую между выхлопным коллектором дизеля и катализатором в потоке выхлопных газов.

В сажевом фильтре развивается высокая температура, при которой частички сажи способны окислиться остаточным кислородом, содержащимся в выхлопных газах. Однако часть сажи не всегда окисляется, и остается в «сажевом фильтре», поэтому программа блока управления периодически переводит двигатель в режим очистки «сажевого фильтра» путём так называемой «постинжекции», то есть впрыска дополнительного количества топлива в цилиндры в конце фазы сгорания с целью поднять температуру газов, и, соответственно, очистить фильтр путём сжигания накопившейся сажи. Стандартом де-факто в конструкциях транспортных дизель-моторов стало наличие турбонаддува (или даже двойного наддува), а в последние годы— и так называемого «интеркулера»— то есть устройства, охлаждающего сжатый турбонагнетателем воздух. Нагнетатель позволил поднять удельные мощностные характеристики массовых дизель-моторов, так как позволяет пропустить за рабочий цикл большее количество воздуха через цилиндры.

В своей основе конструкция дизельного двигателя подобна конструкции бензинового инжекторного двигателя. Однако, аналогичные детали у дизеля обычно тяжелее и более устойчивы к высокому давления сжатия, имеющим место у дизеля. Головки поршней, однако, специально разработаны под особенности сгорания в дизельных двигателях и часто (но не всегда) рассчитаны на повышенную степень сжатия. Кроме того, головки поршней в дизельном двигателе находятся выше верхней плоскости блока цилиндров, когда поршень находится в верхней точке своего хода. Во многих случаях головки поршней содержат в себе камеру сгорания.

КОНСТРУКЦИЯ
Особенности двигателя

Как уже отмечалось, конструкция дизельного двигателя подобна конструкции бензинового двигателя. Однако аналогичные детали у дизеля существенно усилены, чтобы воспринимать более высокие нагрузки — ведь степень сжатия у него намного выше (16-24 единиц против 9-11 у бензинового). Характерная деталь в конструкции дизелей — это поршень. Форма днища поршней у дизелей определяется типом камеры сгорания, поэтому по форме легко определить, какому двигателю принадлежит данный поршень. Во многих случаях днище поршня содержит в себе камеру сгорания. Днища поршней находятся выше верхней плоскости блока цилиндров, когда поршень находится в верхней точке своего хода. Так как воспламенение рабочей смеси осуществляется от сжатия, в дизелях отсутствует система зажигания, хотя свечи могут применяться и на дизеле. Но это не свечи зажигания, а свечи накаливания, которые предназначены для подогрева воздуха в камере сгорания при холодном пуске двигателя.

Технические и экологические показатели автомобильного дизельного двигателя в первую очередь зависят от типа камеры сгорания и системы впрыскивания топлива.

Типы камер сгорания

Форма камеры сгорания значительно влияет на качество процесса смесеобразования, а значит и на мощность и шумность работы двигателя. Камеры сгорания дизельных двигателей разделяются на два основных типа: неразделенные и разделенные.
Несколько лет назад на рынке легкового машиностроения доминировали дизели с разделенными камерами сгорания. Впрыск топлива в этом случае осуществляется не в надпоршневое пространство, а в специальную камеру сгорания, выполненную в головке блока цилиндров. При этом различают два процесса смесеобразования: предкамерный (его еще называют форкамерным) и вихрекамерный.

При форкамерном процессе топливо впрыскивается в специальную предварительную камеру, связанную с цилиндром несколькими небольшими каналами или отверстиями, ударяется об ее стенки и перемешивается с воздухом. Воспламенившись, смесь поступает в основную камеру сгорания, где и сгорает полностью. Сечение каналов подбирается так, чтобы при ходе поршня вверх (сжатие) и вниз (расширение) между цилиндром и форкамерой возникал большой перепад давления, вызывающий течение газов через отверстия с большой скоростью.
Во время вихрекамерного процесса сгорание также начинается в специальной отдельной камере, только выполненной в виде полого шара. В период такта сжатия воздух по соединительному каналу поступает в предкамеру и интенсивно закручивается (образует вихрь) в ней. Впрыснутое в определенный момент топливо хорошо перемешивается с воздухом.
Таким образом, при разделенной камере сгорания происходит как бы двухступенчатое сгорание топлива. Это снижает нагрузку на поршневую группу, а также делает звук работы двигателя более мягким. Недостатком дизельных двигателей с разделенной камерой сгорания являются: увеличение расхода топлива вследствие потерь из-за увеличенной поверхности камеры сгорания, больших потерь на перетекание воздушного заряда в дополнительную камеру и горящей смеси обратно в цилиндр. Кроме того, ухудшаются пусковые качества.

Дизельные двигатели с неразделенной камерой называют также дизелями с непосредственным впрыском. Топливо впрыскивается непосредственно в цилиндр, камера сгорания выполнена в днище поршня. До недавнего времени непосредственный впрыск использовался на низкооборотистых дизелях большого объема (проще говоря, на грузовиках). Хотя такие двигатели экономичнее моторов с разделенными камерами сгорания, их применение на небольших дизелях сдерживалось трудностями организации процесса сгорания, а также повышенными шумом и вибрацией, особенно в режиме разгона.

Система питания дизеля

Важнейшим звеном дизельного двигателя является система топливоподачи, обеспечивающая поступление необходимого количества топлива в нужный момент времени и с заданным давлением в камеру сгорания.

Топливный насос высокого давления (ТНВД), принимая горючее из бака от подкачивающего насоса (низкого давления), в требуемой последовательности поочередно нагнетает нужные порции солярки в индивидуальную магистраль гидромеханической форсунки каждого цилиндра. Такие форсунки открываются исключительно под воздействием высокого давления в топливной магистрали и закрываются при его снижении.

Существует два типа ТНВД: рядные многоплунжерные и распределительного типа. Рядный ТНВД состоит из отдельных секций по числу цилиндров дизеля, каждая из которых имеет гильзу и входящий в нее плунжер, который приводится в движение кулачковым валом, получающим вращение от двигателя. Секции таких механизмов расположены, как правило, в ряд, отсюда и название — рядные ТНВД. Рядные насосы в настоящее время практически не применяются ввиду того, что они не могут обеспечить выполнение современных требований по экологии и шумности. Кроме того, давление впрыска таких насосов зависит от оборотов коленвала.

Распределительные ТНВД создают значительно более высокое давление впрыска топлива, нежели насосы рядные, и обеспечивают выполнение действующих нормативов, регламентирующих токсичность выхлопа. Этот механизм поддерживает нужное давление в системе в зависимости от режима работы двигателя. В распределительных ТНВД система нагнетания имеет один плунжер-распределитель, совершающий поступательное движение для нагнетания топлива и вращательное для распределения топлива по форсункам. Эти насосы компактны, отличаются высокой равномерностью подачи топлива по цилиндрам и отличной работой на высоких оборотах. В то же время они предъявляют очень высокие требования к чистоте и качеству дизтоплива: ведь все их детали смазываются топливом, а зазоры в прецизионных элементах очень малы.

Ужесточение в начале 90-х законодательных экологических требований, предъявляемых к дизелям, заставило моторостроителей интенсивно совершенствовать топливоподачу. Сразу же стало ясно, что с устаревшей механической системой питания эту задачу не решить. Традиционные механические системы впрыска топлива имеют существенный недостаток: давление впрыска зависит от частоты вращения двигателя и нагрузочного режима. Это значит, что при низкой нагрузке давление впрыска падает, в результате топливо при впрыске плохо распыляется, попадая в камеру сгорания слишком крупными каплями, которые оседают на ее внутренних поверхностях. Из-за этого уменьшается КПД сгорания топлива и повышается уровень токсичности отработанных газов.

Кардинально изменить ситуацию могла только оптимизация процесса горения топливовоздушной смеси. Для чего надо заставить весь её объём воспламениться в максимально короткое время. А здесь необходима высокая точность дозы и точность момента впрыскивания. Сделать это можно, только подняв давление впрыска топлива и применив электронное управление процессом топливоподачи. Дело в том, что чем выше давление впрыска, тем лучше качество его распыления, а соответственно – и смешивания с воздухом. В конечном итоге это способствует более полному сгоранию топливовоздушной смеси, а значит и уменьшению вредных веществ в выхлопе. Хорошо, спросите вы, а почему бы не сделать такое же повышенное давление в обычном ТНВД и всей этой системе? Увы, не получится. Потому что есть такое понятие, как «волновое гидравлическое давление». При любом изменении расхода топлива в трубопроводах от ТНВД к форсункам возникают волны давления, «бегающие» по топливопроводу. И чем сильнее давление, тем сильнее эти волны. И если далее повышать давление, то в какой-то момент может произойти обыкновенное разрушение трубопроводов.

В результате были разработаны два новых типа систем питания – в первом форсунку и плунжерный насос объединили в один узел (насос-форсунка), а в другом ТНВД начал работать на общую топливную магистраль (Common Rail), из которой топливо поступает на электромагнитные (или пьезоэлектрические) форсунки и впрыскивается по команде электронного блока управления. Но с принятием Евро 3 и 4 и этого оказалось мало, и в выхлопные системы дизелей внедрили сажевые фильтры и катализаторы.

Насос-форсунка устанавливается в головку блока двигателя для каждого цилиндра. Она приводится в действие от кулачка распределительного вала с помощью толкателя. Магистрали подачи и слива топлива выполнены в виде каналов в головки блока. За счет этого насос-форсунка может развить давление до 2200 бар. Дозированием топлива, сжатого до такой степени и управлением угла опережения впрыска занимается электронный блок управления, выдавая сигналы на запорные электромагнитные или пьезоэлектрические клапаны насос-форсунок. Насос-форсунки могут работать в многоимпульсном режиме (2-4 впрыска за цикл). Это позволяет произвести предварительный впрыск перед основным, подавая в цилиндр сначала небольшую порцию топлива, что смягчает работу мотора и снижает токсичность выхлопа. Недостаток насос-форсунок – зависимость давления впрыска от оборотов двигателя и высокая стоимость данной технологии.

Система питания Common Rail используется в дизелях серийных моделей с 1997 года. Common Rail – это метод впрыска топлива в камеру сгорания под высоким давлением, не зависящим от частоты вращения двигателя или нагрузки. Главное отличие системы Common Rail от классической дизельной системы заключается в том, что ТНВД предназначен только для создания высокого давления в топливной магистрали. Он не выполняет функций дозировки цикловой подачи топлива и регулировки момента впрыска. Система Common Rail состоит из резервуара – аккумулятора высокого давления (иногда его называют рампой), топливного насоса, электронного блока управления (ЭБУ) и комплекта форсунок, соединенных с рампой. В рампе блок управления поддерживает, меняя производительность насоса, постоянное давление на уровне 1600-2000 бар при различных режимах работы двигателя и при любой последовательности впрыска по цилиндрам. Открытием-закрытием форсунок управляет ЭБУ, который рассчитывает оптимальный момент и длительность впрыска, на основании данных целого ряда датчиков – положения педали акселератора, давления в топливной рампе, температурного режима двигателя, его нагрузки и т. п. Форсунки могут быть электромагнитными, либо более современными- пьезоэлектрическими. Главные преимущества пьезоэлектрических форсунок — высокая скорость срабатывания и точность дозирования. Форсунки в дизелях c Common rail могут работать в многоимпульсном режиме: в ходе одного цикла топливо впрыскивается несколько раз – от двух до семи. Сначала поступает крохотная, всего около миллиграмма, доза, которая при сгорании повышает температуру в камере, а следом идет главный «заряд». Для дизеля — двигателя с воспламенением топлива от сжатия — это очень важно, так как при этом давление в камере сгорания нарастает более плавно, без «рыбка». Вследствие этого мотор работает мягче и менее шумно, снижается количество вредных компонентов в выхлопе. Многократная подача топлива за один такт попутно обеспечивает снижение температуры в камере сгорания, что приводит к уменьшению образования окиси азота- одной из наиболее токсичных составляющих выхлопных газов дизеля. Характеристики двигателя с Common Rail во многом зависят от давления впрыска. В системах третьего поколения оно составляет 2000 бар. В ближайшее время в серию будет запущено четвертое поколение Common Rail с давлением впрыска 2500 бар.

А теперь посмотрите обучающие и очень интересное видео о дизельном двигателе.


Турбодизель. Система турбонаддува.

Эффективным средством повышения мощности и гибкости работы дизеля является турбонаддув. Он позволяет подать в цилиндры дополнительное количество воздуха и соответственно увеличить подачу топлива на рабочем цикле, в результате чего увеличивается мощность двигателя. Давление выхлопных газов дизеля в 1,5-2 раза выше, чем у бензинового мотора, что позволяет турбокомпрессору обеспечить эффективный наддув с самых низких оборотов, избежав свойственного бензиновым турбомоторам провала — «турбоямы». Отсутствие дроссельной заслонки в дизеле позволяет обеспечить эффективное наполнение цилиндров на всех оборотах без применения сложной схемы управления турбокомпрессором. На многих автомобилях устанавливается промежуточный охладитель наддуваемого воздуха — интеркулер, позволяющий поднять массовое наполнение цилиндров и на 15-20 % увеличить мощность.

Надув позволяет добиться одинаковой мощности с атмосферным мотором при меньшем рабочем объеме, а значит, снизить массу двигателя. Турбонаддув, помимо всего прочего, служит для автомобиля средством повышения «высотности» двигателя — в высокогорных районах, где атмосферному дизелю не хватает воздуха, наддув оптимизирует сгорание и позволяет уменьшить жесткость работы и потерю мощности. В то же время турбодизель имеет и некоторые недостатки, связанные в основном с надежностью работы турбокомпрессора. Так, ресурс турбокомпрессора существенно меньше ресурса двигателя. Турбокомпрессор предъявляет жесткие требования к качеству моторного масла. Неисправный агрегат может полностью вывести из строя сам двигатель. Кроме того, собственный ресурс турбодизеля несколько ниже такого же атмосферного дизеля из-за большой степени форсирования. Такие двигатели имеют повышенную температуру газов в камере сгорания, и чтобы добиться надежной работы поршня, его приходится охлаждать маслом, подаваемым снизу через специальные форсунки.

Назначение и приборы системы питания дизельного двигателя

 

Какое назначение системы питания дизельного двигателя?

Система питания дизельного двигателя служит для подвода воздуха и топлива в цилиндры двигателя в заданной пропорции и под заданным давлением и отвода отработавших газов из них.

Что входит в устройство системы питания дизельного двигателя автомобиля КамАЗ-5320?

Система питания дизельного двигателя автомобиля КамАЗ-5320 (рис.76) состоит из топливного бака 16; топливного фильтра 18 предварительной (грубой) очистки топлива; топливоподкачивающего насоса 2 с устройством 1 для ручной подкачки топлива; топливного насоса 4 высокого давления; форсунок 6; электромагнитного клапана 8; факельной свечи 10; фильтра 12 для окончательной (тонкой) очистки топлива; топливопроводов низкого 3 и высокого 5 давления; топливоотводящих (дренажных) трубопроводов 9, 11, 14 и 15 с тройником 17; топливопроводов 7 и 13 для подвода топлива соответственно к электромагнитному клапану и топливному насосу; воздушных фильтров; трубопровода для подвода воздуха в цилиндры двигателя и отвода отработавших газов из них; глушители шума выпуска отработавших газов; указателя уровня топлива в топливном баке; регулятора частоты вращения коленчатого вала; педали газа с системой тяг для управления рейкой топливного насоса; автоматической муфты опережения впрыска топлива.

Рис.76. Схема системы питания дизельного двигателя автомобиля КамАЗ-5320.

На отдельных двигателях устанавливают турбокомпрессор для подачи воздуха в цилиндры двигателя под давлением с целью повышения мощности двигателя и снижения токсичности отработавших газов.

Как работает система питания двигателя автомобиля КамАЗ-5320?

Во время работы двигателя топливо из топливного бака поступает по топливопроводу в фильтр предварительной очистки 18 (рис.76), очищается от грубых примесей и воды и топливоподкачивающим насосом под давлением 0,15-0,20 МПа по топливопроводу 3 подается в фильтры тонкой очистки 12, где окончательно очищается. Затем по топливопроводу 13 поступает в топливный насос высокого давления 4, который повышает давление топлива, дозирует его количество для каждого цилиндра в соответствии с порядком работы и нагрузкой двигателя и по топливопроводам 5 высокого давления подает в форсунки 6, которые впрыскивают топливо в цилиндры под давлением 18 МПа. Впрыскнутое топливо смешивается в цилиндре с нагретым при такте сжатия воздухом и испаряется. Образовавшаяся горючая смесь самовоспламеняется и сгорает. Совершается такт рабочего хода, во время которого тепловая энергия преобразуется в механическую, и в виде крутящего момента передается на колеса автомобиля.

Избыточное топливо, а вместе с ним и проникший в систему питания воздух отводятся через перепускной клапан топливного насоса высокого давления и клапан-жиклер фильтра тонкой очистки по дренажным топливопроводам 11 и 14 в топливный бак 16. Топливо, просочившееся в полость пружины форсунки через зазор между корпусом распылителя и иглой, сливается в бак по дренажным топливопроводам 9 и 15 с тройником 17.

Электромагнитный клапан 8 топливопроводом 7 соединен с насосом высокого давления и служит для подачи топлива под давлением 0,06-0,08 МПа к факельным свечам 10, установленным во всех впускных трубопроводах для подогрева воздуха при пуске двигателя в холодное время года.

Система питания других дизельных двигателей устроена и работает так же, если она разделенного типа.

В чем особенности системы питания неразделенного типа и где она применяется?

Система питания дизельных двигателей неразделенного типа применяется на дизельных двухтактных двигателях ЯАЗ-204, ЯАЗ-206. В этой системе насос высокого давления и форсунка объединены в одном при боре, называемом насосом-форсункой, что позволило повысить давление впрыскиваемого топлива до 140 МПа при 2000 об/мин коленчатого вала. Однако работа такого двигателя более жесткая, что снижает срок его службы, в нем отсутствуют топливопроводы высокого давления. Регулятор частоты вращения коленчатого вала двухрежимный. Он устойчиво поддерживает минимальную частоту вращения коленчатого вала на холостом ходу и максимальную – на полных нагрузках двигателя.

***
Проверьте свои знания и ответьте на контрольные вопросы по теме «Система питания дизельного двигателя»

давление, двигатель, дизельный, насос, питание, система, топливный, топливо, топливопровод

Смотрите также:
Купить новый Kia Rio в Москве

Принципиальная схема дизельного двигателя

Контекст 1

… На рисунках 1 и 2 показаны две упрощенные принципиальные схемы четырехтактного дизельного двигателя и изодвигателя. Как и во всех устройствах внутреннего сгорания, воздух сжимается, затем добавляется топливо, сгорание происходит при повышенном давлении и, наконец, энергия извлекается во время расширения горячих дымовых газов. …

Контекст 2

… Чтобы получить высокое максимальное давление цикла и высокую удельную мощность, турбокомпрессор (не показан на Рисунке 1 и Рисунке 2) используется для предварительного сжатия воздуха, как это сделано в дизельные двигатели….

Контекст 3

… в описанной выше процедуре запуска isoengine выполнялся без наддува со скоростью 200 об / мин. Увеличивая подачу топлива и продвигаясь вперед, а также сокращая интервал, в течение которого впускные клапаны открыты, давление в контуре может быть дополнительно увеличено, как показано на Рисунке 10. Углы поворота коленчатого вала, при которых впускные клапаны начинали открываться и закрываться, обозначены ромбы на рисунке 10. …

Контекст 4

… Увеличивая заправку и продвижение и сокращая интервал, в течение которого впускные клапаны открыты, давление в контуре может быть дополнительно увеличено, как показано на Рисунке 10. Указаны углы поворота коленчатого вала, при которых впускные клапаны начинали открываться и закрываться. ромбами на рисунке 10. Две нижние кривые соответствуют тестам 1 и 12 в таблице 2, а две верхние кривые были получены в результате последующих тестов, приведенных в таблице 3. …

Контекст 5

…две нижние кривые соответствуют испытаниям 1 и 12 в таблице 2, в то время как две верхние кривые были получены в результате последующих испытаний, обобщенных в таблице 3. Верхняя кривая на рисунке 10 представляет изобарическое сгорание, поскольку давление в контуре выше, чем давление, достигаемое выхлопом. рекомпрессия газа. …

Контекст 6

… расчеты показали, что в этих условиях топливо будет воздействовать на поршень. На рисунке 11 (вверху) показан ограниченный свободный путь вокруг форсунки в ВМТ.Однако свободный путь становится длиннее, когда поршень начинает двигаться вниз. …

Контекст 7

… свободный путь становится длиннее, когда поршень начинает двигаться вниз. Как показано в нижней части рисунка 11, ситуация значительно улучшается уже через 20 ° после ВМТ. Осмотр поверхности поршня показал образец нагара, который подтвердил интерпретацию попадания топлива на головку поршня. …

Контекст 8

…Форсунки с 4 отверстиями были установлены таким образом, чтобы топливо впрыскивалось к выпускным клапанам, а не к впускным клапанам. Уменьшение диаметра отверстия сопла привело к значительному улучшению характеристик горения, особенно когда топливо впрыскивалось после верхней мертвой точки, как и ожидалось на рисунке 11. Однако было также ясно, что изменения в конструкции сопла все еще не продвинулись далеко. достаточно. …

Контекст 9

… В этих испытаниях использовались форсунки с 4 отверстиями.Некоторые результаты приведены на рисунках 12 и 13, которые показывают давления во всех трех цилиндрах сгорания, давление в выпускном отверстии изокомпрессора и высоту впускного и выпускного клапанов камеры сгорания. …

Контекст 10

… В этих испытаниях использовались форсунки с 4 отверстиями. Некоторые результаты приведены на рисунках 12 и 13, которые показывают давления во всех трех цилиндрах сгорания, давление в выпускном отверстии изокомпрессора и высоту впускного и выпускного клапанов камеры сгорания….

Контекст 11

… давление начинает расти примерно при 50 ° до ВМТ. Это повторное сжатие выхлопных газов, на которое указывают кривые давления в цилиндре перед ВМТ на рисунках 12 и 13, используется для облегчения воспламенения топлива, как описано ранее. Еще одним преимуществом рекомпрессии выхлопных газов является то, что это простой способ добиться некоторой рециркуляции выхлопных газов (EGR). …

Контекст 12

… давление начинает расти примерно при 50 ° до ВМТ.Это повторное сжатие выхлопных газов, на которое указывают кривые давления в цилиндре перед ВМТ на рисунках 12 и 13, используется для облегчения воспламенения топлива, как описано ранее. Еще одним преимуществом рекомпрессии выхлопных газов является то, что это простой способ добиться некоторой рециркуляции выхлопных газов (EGR). …

Контекст 13

… параметры, использованные во время тестов, показанных на рисунках 12 и 13, показаны в таблице 4. В обоих тестах использовался одноимпульсный впрыск топлива (в отличие от результатов теста, показанных ранее на 200 об / мин)….

Контекст 14

… параметры, использованные во время тестов, показанных на рисунках 12 и 13, показаны в таблице 4. В обоих тестах использовался одноимпульсный впрыск топлива (в отличие от результатов теста, показанных ранее на 200 об / мин). …

Контекст 15

… наблюдение также подтверждается упомянутым выше повышенным тепловыделением. На рис. 13 показан кратковременный скачок давления примерно при 13 ° CA после ВМТ. Этот всплеск явно соответствует воспламенению впрыскиваемого топлива вскоре после ВМТ, поскольку его время соответствует прогнозам задержки воспламенения….

Контекст 16

… также было отмечено, что можно было перемещать пик в любом направлении вдоль плато давления, изменяя время впрыска топлива. Уровень задымления снизился еще больше по мере задержки момента зажигания, что дает дополнительную поддержку наблюдениям, касающимся попадания брызг топлива, показанных на рисунке 11. …

Дизельный цикл

— процесс с PV и диаграммой TS

Был изобретен дизельный цикл Рудольфом Дизелем в 1893 году.Он выдвинул идею, с помощью которой мы можем достичь более высокого теплового КПД с высокой степенью сжатия. Все дизельные двигатели работают по этому циклу. В этом цикле в качестве топлива используется дизельное топливо, поскольку его можно сжимать при более высокой степени сжатия. Он также известен как цикл постоянного давления, потому что тепло добавляется в него при постоянном давлении. Он имеет высокий тепловой КПД и степень сжатия (от 11: 1 до 22: 1) по сравнению с циклом Отто.

Двигатель, предложенный Рудольфом, состоит из замкнутого в цилиндре воздуха.Стенки цилиндра идеально не проводят тепло, а вот дно — идеальный проводник тепла. Он имеет горячее и холодное тела и изолирующий колпачок, которые попеременно приводятся в контакт с цилиндром.

Идеальный дизельный цикл состоит из 4 процессов, двух изоэнтропических процессов, одного процесса с постоянным давлением и одного процесса с постоянным объемом.

Процессы

Четыре процесса следующие:

  1. Изэнтропическое (обратимое адиабатическое) сжатие
  2. Подвод тепла при постоянном давлении
  3. Изэнтропическое расширение
  4. Отвод тепла с постоянным объемом.

Также читайте:

Работу этих четырех процессов дизельного цикла можно легко понять с помощью диаграмм P-V и T-S.

Примечание: Для лучшего понимания процесса просмотрите диаграммы P-V и T-S, где это необходимо.

Процесс 1-2: Изэнтропическое сжатие

В этом процессе поршень перемещается из НМТ в ВМТ, и сжатие воздуха происходит изэнтропически. Это означает, что при сжатии энтропия остается постоянной и не происходит оттока тепла от стенок цилиндра (непроводников).Здесь воздух сжимается, поэтому давление увеличивается с P1 до P2, объем уменьшается с V1 до V2, температура увеличивается с T1 до T2, а энтропия остается постоянной (т. Е. S1 = S2).

Процесс 2-3: Добавление тепла постоянного объема

В этом процессе горячее тело поддерживается в контакте с цилиндром, а добавление тепла к воздуху происходит при постоянном давлении. Во время этого процесса поршень на мгновение находится в ВМТ. Давление остается постоянным (т.е. P2 = P3), объем увеличивается от V2 до V3, температура увеличивается от T2 до T3, энтропия увеличивается от S2 до S3.

Процесс 3-4: Изэнтропическое расширение

В этом процессе после добавления тепла расширение воздуха происходит изэнтропически, и работа осуществляется системой. Во время этого процесса поршень движется вниз и достигает НМТ. Давление падает с P3 до P4, объем увеличивается с V3 до V4, температура падает с T3 до T4, а энтропия остается постоянной (т. Е. S3 = S4).

Процесс 4-1: Отвод тепла постоянного объема

В этом процессе поршень на мгновение находится в НМТ, а холодное тело контактирует с цилиндром, и отвод тепла происходит при постоянном объеме.Давление уменьшается с P4 до P1, температура уменьшается с T4 до T1, энтропия уменьшается с S4 до S1, а объем остается постоянным (то есть V4 = V1).

Каким образом дизельный цикл реализован в 4-тактном дизельном двигателе?

Это четыре рабочих процесса дизельного двигателя, которые мы обсуждали. Теперь обсудим, как этот цикл реализован в 4-тактном дизельном двигателе. Когда этот цикл используется в 4-тактном дизельном двигателе, у нас есть 2 дополнительных процесса. Один из них — это процесс всасывания, а другой — процесс выпуска.Для добавления тепла в качестве топлива используется дизельное топливо, которое горит и добавляет тепло.

Давайте подробно обсудим весь процесс

  • 0-1: Процесс всасывания (такт всасывания)
  • 1-2: Процесс изэнтропического сжатия (такт сжатия)
  • 2-3: Подвод тепла с постоянным объемом (зажигание и сгорание топливо)
  • 3-4: Изэнтропическое расширение (рабочий ход)
  • 4-1: Отвод тепла с постоянным объемом (охлаждающая жидкость контактирует со стенками цилиндра)
  • 1-0: Процесс выпуска (ход выпуска)

Процесс 0-1: Процесс всасывания

В этом процессе впускной клапан открывается, и всасывание воздуха происходит при атмосферном давлении.Это называется тактом всасывания.

Процесс 1-2: Изэнтропическое сжатие

Всасываемый воздух теперь сжимается изэнтропически. Из-за сжатия температура воздуха повышается до такого уровня, при котором дизельное топливо воспламеняется. Это называется тактом сжатия.

Процесс 2-3: Добавление тепла при постоянном давлении

В конце сжатия поршень находится в ВМТ, и в это время дизельное топливо впрыскивается в цилиндр через топливную форсунку в распыленной форме.Когда это распыленное дизельное топливо вступает в контакт с горячим сжатым воздухом, оно воспламеняется, и начинается процесс сгорания. Сгорание топлива нагревает двигатель.

Процесс 3-4: Изэнтропическое расширение

В процессе сгорания выделяется большое количество тепла, что создает большую силу на головке поршня, и он перемещается вниз от ВМТ до НМТ. Это силовой ход.

Процесс: 4-1: Отвод тепла постоянного объема

Поршень находится на НМТ, охлаждающая жидкость контактирует со стенками цилиндра, отводит тепло от двигателя и охлаждает.После этого поршень движется вверх.

Процесс: 1-0: Выпускной процесс

В этом процессе поршень перемещается из НМТ в ВМТ, и выпускной клапан открывается. Все сгоревшие газы, оставшиеся в цилиндре, выходят через выпускной клапан. Это такт выпуска.

Так работает четырехтактный компрессорный двигатель.

Сводка

Сводка дизельного цикла с работой, положением поршня и изменениями различных параметров приведены ниже:

S.№ Процесс Работа Положение поршня Изменение параметра
1. 1-2: Изэнтропическое сжатие Сжатие воздуха. BDC в TDC V: уменьшается с V1 до
V2T: увеличивается с T1 до
T2P: увеличивается с P1 до
P2S: энтропия остается постоянной (S1 = S2)
2. 2-3: Тепло постоянного давления
добавление
Тепло добавляется от внешнего источника
(горячее тело)
На мгновение в ВМТ и немного движется вниз. V: увеличивается с V2 до
V3T: увеличивается с T2 до
T3P: остается постоянным (P2 =
P3) S: увеличивается с S2 до
S3
3. 3-4: изэнтропическое расширение Расширение воздуха происходит за счет подвода тепла
.
ВМТ до НМТ V: увеличивается с V3 до
V4T: уменьшается с T3 до
T4P: уменьшается с P3 до
P4S: энтропия остается постоянной (S3 = S4)
4. 4-1: Тепло постоянного объема
отклонение
Тепло отводится в сток. На мгновение при BDC V: Объем остается постоянным (V4 =
V1) T: Уменьшается с T4 до
T1P: Уменьшается с P4 до
P1S: Уменьшается с S4 до
S1

Приложение

Дизельный цикл используется там, где требуется большая мощность при меньшем количестве топлива. В основном используется в двухтактных и четырехтактных дизельных двигателях.

Если вы обнаружите, что что-то отсутствует или неверно, сообщите нам об этом в ваших ценных комментариях.И если вы нашли эту статью информативной, не забывайте комментировать, ставить лайки и делиться.

8 основных частей дизельного двигателя и их функции

Основные детали дизельного двигателя — Рудольф Дизель, возможно, это имя звучит странно для наших. Но он стоит за изобретением дизеля. двигатели. Дизельный двигатель — это двигатель внутреннего сгорания, в котором используется дизельное топливо для осуществления процесса сгорания. Принцип работы дизельный двигатель почти такой же, как бензиновый, но мало разница в шаговом усилии этой машины.

Как насчет компонентов? есть ли другое? из конечно есть разница между дизельными и бензиновыми двигателями. Этот разница возникает из-за того, что дизельный двигатель имеет более жесткий рабочий цикл, поэтому используемый материал также должен быть откорректирован. А вообще 4-х тактный дизель компоненты не такие, как у 4-тактного бензинового двигателя. Если мы обсудим, 4-тактный дизельный двигатель будет состоять из сотен компонентов. Эти компоненты, безусловно, имеют свои обязанности и функции.

Пока мы обсудим только основные части 4-х тактного двигателя. дизельные двигатели и их функции. Главный компонент дизельного двигателя — это все компоненты, которые напрямую связаны с 4-тактным дизельным топливным циклом. В то время как другие компоненты, которые напрямую не связаны, такие как альтенатор или стартер мотор мы обсудим в другой статье. Итак, как называется основная компоненты дизельного двигателя 4-х тактный? см. статью ниже

1.Сборка блока цилиндров


Блок цилиндров — основная составляющая внутреннего сгорания. двигатель как 2-х тактный, так и 4-х тактный.Этот компонент становится основным компонентом разместить различные моторные отсеки, поддерживающие рабочий процесс машина. Как видно на картинке выше, форма блока цилиндров все машины в целом одинаковые, но детали будут разными. Это потому что детали блока цилиндров отрегулированы со всеми компонентами, которые будет придерживаться этого блока.

Блок цилиндров из чугуна с высокой степенью прочности. точность. Обычно на блоке цилиндр состоит из нескольких компонентов;


  • Цилиндр / основная футеровка.Этот компонент будет служить местом вверх и вниз по поршню.Компоненты изготовлены из сплава железа и алюминия. расположен внутри блока цилиндров с помощью прессового метода, поэтому его будет сложно отсоединить.
  • Водяная рубашка. Водяная рубашка — это оболочка охлаждающей воды, расположенная внутри блока двигателя. Цель процесса охлаждения двигателя выполнена. дырчатая водяная рубашка внутри блока цилиндров, которая окружает подкладку.
  • Трубопроводы подачи масла. Масляное отверстие на блоке цилиндров служит для создать маслопровод двигателя от головки блока цилиндров до картера.Эта дыра будет поддерживать процесс циркуляции моторного масла ко всем частям дизельного двигателя.

2. Сборка головки блока цилиндров


Второй компонентный блок расположен в верхней части двигателя. Как и блок цилиндров, этот элемент также выполнен из литого материала. Сейчас вариант с алюминиевой головкой цилиндра, потому что он легче. и сильнее. Этот блок состоит из клапана и пружины, распределительного вала, коромысла и камера сгорания.


  • Клапан и пружина. Этот компонент становится дверью, которая откроет и закроет впускной и выпускной каналы в камере сгорания. Пока пружина будет держать клапан закрытым.
  • Распредвал. Распредвал — это деталь, имеющая несколько кулачков, у этого кулачка есть грибок, чтобы нажимать на клапан. После нажатия клапана канал впускной или выпускной порт откроется.
  • Коромысло. Этот компонент будет нажимать на клапан, когда касается верхней части коромысла. Так может открыться входной / выходной канал.В целом коромысло имеет систему регулировки зазора клапана, ручную или автоматическую (Гидравлический регулятор ресниц).
  • Камера сгорания. Камера сгорания — это небольшое пространство используется для сжигания. в результате возникает вспышка огня, которая толкает поршень вниз. Обычно эта камера сгорания встречается в дизельных двигателях с непрямым впрыском.

3.Поршень и шатун


Поршень имеет функцию регулировки громкости внутри цилиндр.почему объем цилиндра нужно регулировать? это так что может иметь место рабочий процесс 4-х тактного двигателя. В этом случае, когда поршень движется вниз, объем цилиндра увеличивается, а когда поршень движется вверх, объем цилиндра сжимается. Пока шатун служит для продолжения движения поршня вверх и вниз к маховику. В основном На поршне есть три основные части, а именно;


  • Кольцо компрессионное. Эти кольца эластичны по отношению к предотвратить возникновение утечек воздуха во время такта сжатия.Как это Кольцо работает, чтобы закрыть зазор между стенкой поршня и основной футеровкой.
  • Масляное кольцо. Кольцо, отпечатанное под компрессионным кольцом, служит для не допускать попадания моторного масла в камеру сгорания.
  • Пальцы поршневые. Штифт, расположенный внутри поршня для соединения поршень с шатуном. Этот штифт трубчатый, когда он подсоединяется к На малом конце он будет работать как шарнир.

4. Коленчатый вал

Коленчатый вал компонент из чугуна, который используется для поворота поршня вверх и вниз во вращательное движение.Принцип работы коленчатого вала аналогичен при использовании велосипед. Поскольку это связано с давлением поршня, коленчатый вал не должен быть податливым или ломаться под давлением поршня. Для этот компонент изготовлен из специального сплава железа, обладающего высокой прочностью и анти-стойкость. Некоторые детали на коленчатом валу есть;

Шатун шатунный. Штифт кривошипа — это штифт, который подключается к большому конец на шатуне.

Кривошипный журнал. В то время как шейка кривошипа представляет собой штифт, который служит как вал на коленвале для того, чтобы крутить.Журналирование кривошипа будет крепится к блоку цилиндров.

Весовой баланс. Этот компонент расположен напротив кривошипа. штифт, его функция как противовес, а также для слива масла со всего внутри машины.

5. Масляный поддон

Масляный поддон (Картер) — это специальная ванна, служащая для размещения машинное масло. Хотя этот компонент служил только контейнером с моторным маслом, тоже нельзя сделать по неосторожности. Обычно эти компоненты изготавливаются из тонких железо похоже на цинк, но в некоторых автомобилях используются более толстые материалы.

6. Сборка цепи привода ГРМ


Цепь ГРМ входит в систему клапанного механизма, его функция связывать вращение коленчатого и распределительного валов с определенным угол. Компонент этой цепи расположен на передней части двигателя. Этот цепь соединит звездочку от коленчатого вала со звездочкой распредвала.

7. Маховик


Маховик изначально служит для уравновешивания оборотов двигателя.Этот компонент изготовлен из твердого железа, способного сохранять крутящий момент, поэтому этот Компонент может уравновешивать частоту вращения двигателя.

Кроме того, маховик также служит для привода двигателя, это можно увидеть снаружи маховика с множеством шестерен. Снаряжение будет быть соединенным с шестерней стартера для запуска двигателя.

8. Сборка топливной системы

Этот компонент состоит из топливного бака и форсунки. Дизель топливная система служит для подачи некоторого количества дизельного топлива в камеру сгорания. камера во время инсульта.В дизельном топливе есть два вида топливных систем. двигатели, а именно обычные системы и системы Common Rail. Преимущества дизеля двигатели, использующие Common Rail, более эффективны и экономичны. Это потому что система Common Rail имеет компьютеризированное управление, поэтому расчеты можно сделать точно.

Вышеупомянутые компоненты очень влияют на успех рабочий цикл дизеля. Если есть повреждение любого из вышеперечисленных компонентов, будет нарушен рабочий процесс дизеля.

Вероятно, достаточно, чтобы здесь обсудить основные части дизельные двигатели и их функции, могут быть полезны.

Диаграмма высококачественного и эффективного дизельного двигателя

для транспортных средств

Поддержите свои двигатели и оборудование в их оптимальном рабочем состоянии и эффективности с помощью выдающейся диаграммы дизельного двигателя , доступной на Alibaba.com в виде заманчивых предложений. Они входят в обширную коллекцию, которая подходит для разных пользователей, таких как диаграмма дизельного двигателя продавцов и дистрибьюторов, а также для частных пользователей.Схема дизельного двигателя оснащена современными передовыми технологиями для решения различных проблем с оборудованием и двигателями, которые могут снизить производительность.

Эти дизельные двигатели , схема собраны с использованием тщательно отобранных материалов для обеспечения высочайшего уровня эффективности при сохранении долговечности. Вы обнаружите, что существует широкий выбор этой схемы дизельного двигателя , чтобы все пользователи могли найти свои подходящие типы продуктов и удовлетворить различные требования и спецификации в соответствии с потребностями.Наивысшее качество продукции гарантируется сертифицированными поставщиками и дистрибьюторами дизельных двигателей , которые тщательно проверяются перед утверждением.

Забудьте о любых сбоях, вызванных отказом двигателя, благодаря идеально подходящей схеме дизельного двигателя . Их материалы, дизайн и стили делают их очень эффективными. Схема дизельного двигателя обладает высокой устойчивостью к экстремальным условиям окружающей среды и эксплуатации, таким как высокие температуры и влажность.На Alibaba.com, диаграмма дизельного двигателя Дистрибьюторы всегда готовы подсказать покупателям, как их использовать оптимальным образом. Несмотря на свои удивительные и полезные свойства, они очень доступны по цене.

Изучите Alibaba.com и оцените широкую диаграмму дизельных двигателей диапазонов, чтобы вы могли выбрать те, которые лучше всего подходят вашим требованиям и бюджету. Размещение индивидуальных заказов также возможно в зависимости от количества и конкретных типов. Найдите наиболее подходящую схему дизельного двигателя и обсудите более индивидуальные и индивидуальные варианты.

Диаграмма давление-объем (pV) и то, как работа выполняется в ДВС — x-engineer.org

Двигатель внутреннего сгорания — это тепловой двигатель . Принцип его работы основан на изменении давления и объема внутри цилиндров двигателя. Все тепловые двигатели характеризуются диаграммой давление-объем , также известной как диаграмма pV , которая в основном показывает изменение давления в цилиндре в зависимости от его объема для полного цикла двигателя.

Также работа , производимая двигателем внутреннего сгорания, напрямую зависит от изменения давления и объема внутри цилиндра.

К концу этого руководства читатель должен уметь:

  • понять значение диаграммы pV
  • как нарисовать диаграмму pV для 4-тактного двигателя внутреннего сгорания
  • при впуске и выпуске клапаны приводятся в действие во время цикла двигателя
  • , когда зажигание / впрыск производится во время цикла двигателя
  • как работа создается двигателем внутреннего сгорания
  • какая разница между указанным и работа тормоза
  • каков механический КПД двигателя

Давайте начнем с рассмотрения pV-диаграммы четырехтактного атмосферного двигателя внутреннего сгорания.

Изображение: диаграмма давление-объем (pV) для типичного 4-тактного ДВС

где:

S — ход поршня
V c — зазорный объем
V d — смещенный (рабочий) объем
p 0 — атмосферное давление
W — работа
ВМТ — верхняя мертвая точка
НМТ — нижняя мертвая точка
IV — впускной клапан
EV — выпускной клапан
IVO — открытие впускного клапана
IVC — закрытие впускного клапана
EVO — открытие выпускного клапана
EVC — закрытие выпускного клапана
IGN (INJ) — зажигание (впрыск)

Диаграмма давление-объем (pV) построена путем измерения давления внутри цилиндра и его значения в зависимости от угла поворота коленчатого вала в течение всего цикл двигателя (720 °).

Давайте посмотрим, что происходит в цилиндре во время каждого хода поршня, как изменяются давление и объем внутри цилиндра.

Обратите внимание, что синхронизация впускных и выпускных клапанов имеет опережение и задержку относительно положения поршня. Например, впускной клапан открывается во время такта выпуска поршня и закрывается во время такта сжатия. В то же время, когда начинается такт впуска, выпускной клапан еще некоторое время открыт.Открытие выпускного клапана происходит до завершения рабочего хода.

ВПУСК (a-b)

Цикл двигателя начинается в точке a . Впускной клапан уже открыт, и поршень движется от ВМТ к НМТ. Объем постоянно увеличивается по мере того, как поршень перемещается по длине хода. Максимальный объем достигается, когда поршень находится в НМТ. Давление ниже атмосферного на протяжении всего хода, потому что движение поршня создает объем, а воздух втягивается внутрь цилиндра из-за эффекта вакуума.

СЖАТИЕ (b-c)

После того, как поршень прошел НМТ, начинается такт сжатия. В этой фазе объем начинает уменьшаться, а давление увеличиваться. Требуется некоторое время, пока давление в цилиндре не превысит атмосферное, чтобы впускной клапан оставался открытым даже после того, как поршень пройдет НМТ. По мере того, как поршень приближается к ВМТ, давление постепенно увеличивается. Примерно за 25 ° до ВМТ срабатывает зажигание, и давление быстро повышается до максимального.

МОЩНОСТЬ (c-e)

После события зажигания / впрыска давление в цилиндре резко возрастает, пока не достигнет максимальных значений p max . Значение максимального давления зависит от типа двигателя, на каком топливе он используется. Для типичного двигателя легкового автомобиля максимальное давление в цилиндре может составлять около 120 бар (бензин) или 180 бар (дизель). Рабочий ход начинается, когда поршень движется от ВМТ к НМТ. Высокое давление в цилиндре толкает поршень, поэтому объем увеличивается, а давление начинает постепенно падать.

ВЫХЛОП (e-a)

После рабочего хода поршень снова находится в НМТ. Объем в цилиндре снова на максимальном значении, а давление около минимального (атмосферное давление). Поршень начинает двигаться в сторону ВМТ и выталкивает сгоревшие газы из цилиндра.

Как видите, давление и объем внутри цилиндров двигателя постоянно меняются. Мы увидим, что работа, производимая ДВС, зависит от изменений давления и объема.

Работа Вт [Дж] — это произведение силы F [Н] , которая толкает поршень, и смещения, которое в нашем случае представляет собой ход S [м] .

\ [W = F \ cdot S \ tag {1} \]

Мы знаем, что давление — это сила, разделенная на площадь, поэтому:

\ [F = p \ cdot A_p \ tag {2} \]

, где p [ Па] давление внутри цилиндра и A p 2 ] — площадь поршня.

Замена (2) в (1) дает:

\ [W = p \ cdot A_p \ cdot S \ tag {3} \]

Мы знаем, что умножая расстояние на площадь, мы получаем объем, следовательно:

\ [W = p \ cdot V \ tag {4} \]

Это мгновенная работа , произведенная в цилиндре при определенном давлении и объеме.Чтобы определить работу для полного цикла двигателя, нам нужно интегрировать мгновенную работу:

\ [W = \ int F \ cdot dx = \ int p \ cdot A_p \ cdot dx \ tag {5} \]

, где x ход поршня.

Произведение между ходом поршня и площадью поршня дает дифференциальный объем dV , смещенный поршнем:

\ [dV = A_p \ cdot dx \ tag {6} \]

Замена (6) в (5 ) дает работу , произведенную в цилиндре за полный цикл :

\ [\ bbox [# FFFF9D] {W = \ int p \ cdot dV} \ tag {7} \]

Поскольку подавляющее большинство Если двигатель внутреннего сгорания имеет несколько цилиндров, мы собираемся ввести более подходящий параметр для количественной оценки работы, которым является удельная работа Вт [Дж / кг] .

\ [w = \ frac {W} {m} \ tag {8} \]

где м [кг] — масса топливовоздушной смеси внутри цилиндров за полный цикл.

Мы можем также определить удельный объем v [m 3 / кг] как:

\ [v = \ frac {V} {m} \ tag {9} \]

Производная от удельного объем будет:

\ [dv = \ frac {1} {m} \ cdot dV \ tag {10} \]

, откуда мы можем записать:

\ [dV = m \ cdot dv \ tag {11} \]

Замена (7) в (8) дает:

\ [w = \ frac {1} {m} \ int p \ cdot dV \ tag {12} \]

Из (11) и (12) мы получаем математическое выражение удельной работы для полного цикла двигателя:

\ [\ bbox [# FFFF9D] {w = \ int p \ cdot dv} \]

Работа, производимая внутри цилиндров двигателя, называется , указана удельная работа , w i [Дж / кг] .Что мы получаем на коленчатом валу, так это удельная работа тормоза w b [Дж / кг] . Это называется «тормозом», потому что при испытании двигателей на испытательном стенде они подключаются к тормозному устройству (гидравлическому или электрическому), которое имитирует нагрузку.

Чтобы получить работу тормоза, мы должны вычесть из указанной работы все потери двигателя. Потери связаны с внутренним трением и вспомогательными устройствами, которые требуют мощности от двигателя (масляный насос, водяной насос, нагнетатель, компрессор кондиционера, генератор переменного тока и т. Д.). Эти потери равны удельной работе на трение w f [Дж / кг] .

\ [w_b = w_i — w_f \]

Посмотрев на указанную выше диаграмму давление-объем (pV), мы можем увидеть, что есть две отдельные области:

  • верхняя область, образованная во время тактов сжатия и мощности ( + W)
  • нижняя область, образующаяся во время тактов выпуска и впуска (-W), также называемая насосная работа

В зависимости от значения давления всасывания рабочая область нагнетания может быть отрицательной или положительной.Для атмосферных двигателей насосная работа отрицательна, потому что она использует энергию двигателя для выталкивания выхлопных газов из цилиндров и всасывания свежего воздуха во время впуска.

Для бензиновых атмосферных двигателей из-за дросселирования всасываемого воздуха насосные потери выше и максимальны на холостом ходу. Дизельные двигатели более эффективны, чем бензиновые, потому что на впуске нет дроссельной заслонки, а нагрузка регулируется за счет впрыска топлива.

Если разделить удельный крутящий момент тормоза на указанный удельный крутящий момент, мы получим механический КПД двигателя η м [-] :

\ [\ bbox [# FFFF9D] {\ eta_m = \ frac {w_b} {w_i}} \]

Для большинства двигателей механический КПД составляет около 80-85% при полной нагрузке (полностью открытая дроссельная заслонка) и падает до нуля на холостом ходу, когда весь крутящий момент двигателя используется для поддержания холостого хода. скорость, а не движущая сила.

По любым вопросам, наблюдениям и запросам относительно этой статьи используйте форму комментариев ниже.

Не забывайте ставить лайки, делиться и подписываться!

Глава 3c — Первый закон — Закрытые системы

Глава 3c — Первый закон — Закрытые системы — Дизельные двигатели (обновлено 19.03.2013)

Глава 3: Первый закон термодинамики для Закрытые системы

c) Дизельный цикл воздушного стандарта (Компрессионное зажигание) Двигатель

The Air Стандартный дизельный цикл — идеальный цикл для Компрессионное зажигание (CI) поршневые двигатели, впервые предложенные Рудольфом Дизель более 100 лет назад.Следующая ссылка на Kruse Technology Partnership описывает четырехтактный дизельный цикл работа с коротким история Рудольфа Дизеля. Четырехтактный дизельный двигатель обычно используется в автомобильных системах, тогда как более крупные морские системы обычно используйте двухтактный дизельный цикл . Еще раз у нас есть отличная анимация, созданная Matt Кевени , представляя работу четырехтактный дизельный цикл .

Фактический цикл CI чрезвычайно сложен, поэтому в при первоначальном анализе мы используем идеальное «стандартное» допущение, в котором рабочее тело представляет собой фиксированную массу воздуха, испытывающего полный цикл, который рассматривается как идеальный газ.Все процессы идеальны, горение заменяется добавлением тепла к воздух, а выхлоп заменяется процессом отвода тепла, который восстанавливает воздух в исходное состояние.

Идеальный дизельный двигатель стандартного воздушного отдельные процессы, каждый из которых может быть проанализирован отдельно, как показан в P-V диаграммы ниже. Два из четырех процессов цикла адиабатические процессы (адиабатический = отсутствие передачи тепла), таким образом, прежде чем мы можем продолжить, нам нужно разработать уравнения для идеального газа адиабатический процесс следующим образом:

The Адиабатический процесс идеального газа (Q = 0)

Результатом анализа являются следующие три основных форм, представляющих адиабатический процесс:


где k — коэффициент теплоемкостей и имеет номинальное значение 1.4 в 300К по воздуху.

Процесс 1-2 — это процесс адиабатического сжатия. Таким образом, температура воздуха увеличивается во время сжатия. процесс, а при большой степени сжатия (обычно> 16: 1) он достигнет температуры воспламенения впрыскиваемого топлива. Таким образом данный условия в состоянии 1 и степень сжатия двигателя, в для определения давления и температуры в состоянии 2 (при конец процесса адиабатического сжатия) имеем:

Работа W 1-2 , необходимая для сжатия газа показана как область под кривой P-V и оценивается как следует.

Альтернативный подход с использованием уравнения энергии использует преимущество адиабатического процесса (Q 1-2 = 0) приводит к гораздо более простому процессу:


(спасибо студентке Николь Блэкмор за то, что она рассказала мне об этой альтернативе подход)

Во время процесса 2-3 топливо впрыскивается и сгорает. и это представлено процессом расширения при постоянном давлении. В состояние 3 («прекращение подачи топлива») процесс расширения продолжается адиабатически с понижением температуры до тех пор, пока расширение не станет равным полный.

Процесс 3-4, таким образом, представляет собой процесс адиабатического расширения. Общий объем работы расширения составляет W exp . = (Ш 2-3 + Ш 3-4 ) и отображается как область под P-V диаграмму и анализируется следующим образом:

Наконец, процесс 4-1 представляет постоянный объем процесс отвода тепла. В реальном дизельном двигателе газ просто выпускается из цилиндра, и вводится свежий заряд воздуха.

Чистая работа W net , выполненная за цикл, составляет определяется по формуле: W net = (W exp + W 1-2 ), где, как и раньше, работа сжатия W 1-2 отрицательна (работа выполнена по системе ).

В дизельном двигателе Air-Standard вход Q в происходит за счет сжигания топлива, которое впрыскивается контролируемым образом, в идеале приводящий к процессу расширения при постоянном давлении 2-3, так как показано ниже. При максимальном объеме (нижняя мертвая точка) сгоревшие газы просто истощаются и заменяются свежим зарядом воздуха. Это представлен эквивалентным процессом отвода тепла с постоянным объемом Q из = -Q 4-1 . Оба процесса анализируются следующим образом:

На этом этапе мы можем удобно определить КПД двигателя по тепловому потоку:

__________________________________________________________________________

В этом разделе резюмируются следующие проблемы:

Задача 3.4 А поршневой цилиндр без трения содержит 0,2 кг воздуха при 100 кПа. и 27 ° С. Теперь воздух медленно сжимается в соответствии с соотношением P V k = константа, где k = 1,4, до достижения конечного температура 77 ° C.

  • a) Набросок P-V диаграмма процесса относительно соответствующей константы температурными линиями и обозначьте проделанную работу на этой диаграмме.

  • б) Использование основного определение границы выполненной работы определение границы работы выполнено в процессе [-7.18 кДж].

  • c) Используя уравнение энергии, определите тепла. передано в процессе [0 кДж] и убедитесь, что процесс находится в факт адиабатический.

Производное все уравнения использовались начиная с с основным уравнением энергии для непроточной системы уравнение для изменения внутренней энергии идеального газа (Δu) основное уравнение для выполненной граничной работы и уравнение состояния идеального газа [ P.V = m.R.T ]. Использовать значения удельной теплоемкости определены при 300К для всего процесс.

Проблема 3.5 Учитывать ход расширения только типичный дизельный двигатель Air Standard, имеющий степень сжатия коэффициент 20 и коэффициент отсечки 2. В начале процесса (впрыск топлива) начальная температура 627 ° C, а воздух расширяется при постоянном давлении 6,2 МПа до отсечки (объемное соотношение 2: 1). Впоследствии воздух адиабатически расширяется (без теплопередачи). пока не достигнет максимальной громкости.

  • a) Нарисуйте это процесс на P-v диаграмма, четко показывающая все три состояния.Укажите на схеме общая работа, проделанная в течение всего процесса расширения.

  • б) Определите температуры, достигнутые в конце постоянного давления (топливо впрыск) процесс [1800K], а также в конце процесса расширения [830K], и нарисуйте три соответствующие линии постоянной температуры на P-v диаграмма.

  • c) Определите общая работа, выполненная во время такта расширения [1087 кДж / кг].

  • г) Определите общее количество тепла, подаваемого в воздух. во время такта расширения [1028 кДж / кг].

Вывести все используемые уравнения исходя из уравнения состояния идеального газа и адиабатического процесса соотношения, основное уравнение энергии для замкнутой системы, внутренняя энергия и энтальпия изменяют соотношения для идеального газа, и базовое определение граничной работы, выполняемой системой (при необходимости). Используйте значения удельной теплоемкости, определенные при 1000K для всего процесс расширения, полученный из таблицы Specific Теплоемкость воздуха 903 14.

Решенная проблема 3.6 Идеальный дизельный двигатель, отвечающий требованиям воздушного стандарта, имеет степень сжатия 18 и степень отсечки 2. В начале процесса сжатия рабочая жидкость находится при 100 кПа, 27 ° C (300 К). Определите температуру и давление воздуха в конце каждого процесса, чистый объем работы за цикл [кДж / кг] и термический КПД.

Обратите внимание, что номинальные значения удельной теплоемкости для воздуха при 300K используются C P = 1,00 кДж / кг.K, C v = 0.717 кДж / кг · K ,, и k = 1,4. Однако все они являются функциями температура, и с чрезвычайно высоким температурным диапазоном при работе с дизельными двигателями можно получить значительные ошибки. Один подход (который мы примем в этом примере) заключается в использовании типичного средняя температура на протяжении всего цикла.

Подход к решению:

Первый шаг — нарисовать диаграмму, представляющую проблема, включая всю необходимую информацию. Мы замечаем, что не указаны ни объем, ни масса, поэтому диаграмма и решение будут быть в конкретных количествах.Самая полезная диаграмма для тепловой двигатель P-v схема полного цикла:

Следующим шагом является определение рабочей жидкости и определитесь с основными уравнениями или таблицами для использования. В этом случае рабочая жидкость — воздух, и мы решили использовать среднюю температура 900K на протяжении всего цикла для определения удельной теплоемкости значения емкости представлены в таблице Удельные теплоемкости воздуха 903 14.

Теперь мы проходим все четыре процесса, чтобы определять температуру и давление в конце каждого процесса.

Обратите внимание, что альтернативный метод оценки давление P 2 — это просто использовать уравнение состояния идеального газа, как показано ниже:

Любой из подходов удовлетворителен — выберите тот, который вам удобнее. Теперь продолжим с топливом процесс постоянного давления впрыска:



Обратите внимание, что даже если проблема запрашивает «net производительность за цикл »мы рассчитали только тепло в и разогреть.В случае с дизельным двигателем намного проще оценить значения тепла, и мы можем легко получить чистую работу из энергетический баланс за полный цикл выглядит следующим образом:

Вы можете удивиться нереально высокой температуре полученная эффективность. В этом идеализированном анализе мы проигнорировали многие эффекты потерь, которые существуют в практических тепловых двигателях. Мы начнем понять некоторые из этих механизмов потерь, когда мы изучаем Второй закон in Глава 5 .

______________________________________________________________________________

К части d) Закона Первый закон — Цикловые двигатели Отто

______________________________________________________________________________________


Инженерная термодинамика, Израиль Уриэли под лицензией Creative Общедоступное авторское право — Некоммерческое использование — Совместное использование 3.0 Соединенные Штаты Лицензия

Дизельный цикл

— Дизельный двигатель | Определение

Дизельный цикл — pV, Ts диаграмма

PV диаграмма идеального дизельного цикла

Дизельные циклы часто наносятся на диаграмму давление-объем (диаграмма pV) и диаграмма температуры-энтропии (диаграмма Ts).

На диаграмме давление-объем изобарный процесс следует изобарной линии для газа (горизонтальные линии), изохорный процесс следует изохорной линии для газа (вертикальная линия), адиабатические процессы протекают между ними. линии и область, ограниченная полной траекторией цикла, представляют общей работы , которая может быть выполнена в течение одного цикла.

Диаграмма температура-энтропия (Ts диаграмма), в которой термодинамическое состояние указано точкой на графике с удельной энтропией (s) в качестве горизонтальной оси и абсолютной температурой (T) в качестве вертикальной оси. Ts-диаграммы — полезный и распространенный инструмент, особенно потому, что они помогают визуализировать теплопередачу во время процесса. Для обратимых (идеальных) процессов площадь под кривой T-s процесса — это тепла, переданного системе в течение этого процесса .

Четырехтактный дизельный двигатель

Дизельные двигатели могут быть двухтактными или четырехтактными. Четырехтактный дизельный двигатель — это двигатель внутреннего сгорания (ВС), в котором поршень совершает четыре отдельных хода при вращении коленчатого вала. Ход означает полный ход поршня вместе с цилиндром в любом направлении. Следовательно, каждый ход не соответствует одному термодинамическому процессу, описанному в главе Цикл дизельного двигателя — Процессы.

Четырехтактный двигатель состоит из:

  • Дизельный двигатель по принципу действия аналогичен бензиновому.Свеча зажигания воспламеняет двигатель Отто на этом снимке, а не само сжатие. Четырехтактный двигатель — двигатель Отто
    Источник: wikipedia.org, Собственная работа Зефириса, CC BY-SA 3.0

    Ход впуска — Поршень перемещается от верхней мертвой точки (ВМТ) к нижней мертвой точке (НМТ) , и цикл проходит 0 → 1. В этом такте впускной клапан открыт, в то время как поршень втягивает воздух (без топлива) в цилиндр, создавая вакуумное давление в цилиндре посредством его нисходящего движения.

  • Ход сжатия — Поршень перемещается от нижней мертвой точки (НМТ) к верхней мертвой точке (ВМТ), и цикл проходит точки 1 → 2. И впускной, и выпускной клапаны закрыты в этом такте, что приводит к адиабатическому сжатию воздуха (то есть без передачи тепла в окружающую среду или из нее). Во время этого сжатия объем уменьшается, давление и температура повышаются. В конце этого хода топливо впрыскивается и сгорает в сжатом горячем воздухе.В конце этого хода коленчатый вал совершил полный оборот на 360 градусов.
  • Рабочий ход — Поршень движется от верхней мертвой точки (ВМТ) к нижней мертвой точке (НМТ), и цикл проходит через точки 2 → 3 → 4. В этом ходе и впуск, и выпуск клапаны закрыты. В начале рабочего такта происходит почти изобарическое горение между 2 и 3. В этом интервале давление остается постоянным, поскольку поршень опускается, а объем увеличивается.В точке 3 впрыск топлива и сгорание завершены, и в цилиндре содержится газ с более высокой температурой, чем в точке 2. Между 3 и 4 этот горячий газ расширяется снова приблизительно адиабатически. В этом ходе поршень движется по направлению к коленчатому валу, объем увеличивается, и работа выполняется за счет газа на поршне.
  • Ход выпуска. Поршень перемещается из нижней мертвой точки (НМТ) в верхнюю мертвую точку (ВМТ), и цикл проходит 4 → 1 → 0. Выпускной клапан открыт в этом такте, в то время как поршень вытягивает отработавшие газы из камера.В конце этого хода коленчатый вал совершает второй полный оборот на 360 градусов.

Обратите внимание: в идеальном случае адиабатическое расширение должно продолжаться до тех пор, пока давление не упадет до окружающего воздуха. Это повысило бы тепловой КПД такого двигателя, но это также вызывает практические трудности. Просто двигатель должен был быть намного больше.

Сравнение фактического и идеального дизельного цикла

В этой статье показан идеальный дизельный цикл , в котором есть много предположений, которые отличаются от реального дизельного цикла .Основные различия между реальными и идеальными дизельными двигателями показаны на рисунке. В действительности идеального цикла не существует, и с каждым процессом связано много потерь. Для фактического цикла форма pV-диаграммы подобна идеальной, но площадь (работа), ограниченная pV-диаграммой, всегда меньше идеального значения. Идеальный дизельный цикл основан на следующих предположениях:

  • Замкнутый цикл : Самая большая разница между двумя диаграммами заключается в упрощении тактов впуска и выпуска в идеальном цикле.Во время такта выпуска тепло Q out выбрасывается в окружающую среду (в реальном двигателе газ выходит из двигателя и заменяется новой смесью воздуха и топлива).
  • Добавление изобарического тепла . В реальных двигателях добавление тепла никогда не бывает изобарическим.
  • Нет теплопередачи
    • Сжатие — газ адиабатически сжимается из состояния 1 в состояние 2. В реальных двигателях всегда есть некоторые недостатки, которые снижают термический КПД.
    • Расширение. Газ адиабатически расширяется из состояния 3 в состояние 4.
  • Полное сгорание смеси.
  • Насосные работы не выполняются . Насосная работа — это разница между работой, выполняемой во время такта выпуска и такта впуска. В реальных циклах существует разница между давлением на выходе и на входе.
  • Без потерь при продувке . Потеря продувки вызвана преждевременным открытием выпускных клапанов. Это приводит к потере выходной мощности во время хода расширения.
  • Отсутствие потери от газовых потоков . Утечка сжатых газов приводит к утечке через поршневые кольца и другие щели.
  • Без потерь на трение .

Эти упрощающие допущения и потери приводят к тому, что замкнутая область (работа) pV-диаграммы для реального двигателя значительно меньше, чем площадь (работа), заключенная pV-диаграммой идеального цикла. Другими словами, идеальный цикл двигателя будет переоценивать сеть, и, если двигатели работают с одинаковой скоростью, фактический двигатель производит большую мощность примерно на 20% (так же, как в случае двигателя Отто).

Степень сжатия — двигатель Otto

Степень сжатия , CR определяется как соотношение объема в нижней мертвой точке и объема в верхней мертвой точке. Это ключевая характеристика многих двигателей внутреннего сгорания. В следующем разделе будет показано, что степень сжатия определяет тепловой КПД используемого термодинамического цикла двигателя внутреннего сгорания. Желательно иметь высокую степень сжатия, поскольку это позволяет двигателю достичь более высокого теплового КПД.

Например, предположим, что цикл Отто со степенью сжатия CR = 10: 1. Объем камеры составляет 500 см³ = 500 × 10 -6 м 3 (0,5 л) до такта сжатия. Для этого двигателя a ll требуемых объемов известны:

  • V 1 = V 4 = V max = 500 × 10 -6 м 3 (0,5 л)
  • V 2 = V 3 = V min = V max / CR = 55,56 × 10 -6 м 3

Обратите внимание, что (V max — V min ) x количество цилиндров = общий рабочий объем двигателя.

Примеры степеней сжатия — бензин и дизельное топливо

  • Степень сжатия в бензиновом двигателе обычно не будет намного выше 10: 1 из-за потенциальной детонации двигателя (самовоспламенения) и не ниже 6: 1 .
  • Subaru Impreza WRX с турбонаддувом имеет степень сжатия 8,0: 1 . Как правило, двигатели с турбонаддувом или наддувом уже имеют сжатый воздух на входе. Поэтому они обычно изготавливаются с более низкой степенью сжатия.
  • Стандартный двигатель Honda S2000 (F22C1) имеет степень сжатия 11,1: 1 .
  • Некоторые атмосферные двигатели спортивных автомобилей могут иметь степень сжатия до 12,5: 1 (например, Ferrari 458 Italia).
  • В 2012 году Mazda выпустила новые бензиновые двигатели под торговой маркой SkyActiv со степенью сжатия 14: 1 . Остаточный газ сокращается за счет использования выхлопных систем двигателя 4-2-1, реализации полости поршня и оптимизации впрыска топлива для снижения риска детонации двигателя.
  • Дизельные двигатели имеют степень сжатия, которая обычно превышает 14: 1, и степень сжатия более 22: 1 также является обычным явлением.

Тепловой КПД для дизельного цикла

В целом тепловой КПД , η th , любого теплового двигателя определяется как отношение выполняемой работы, Вт , к погонной энергии при высокой температуре Q H .

Тепловой КПД , η th , представляет собой долю тепла , Q H , преобразовано в работу .Поскольку энергия сохраняется в соответствии с первым законом термодинамики и энергия не может быть полностью преобразована для работы, подвод тепла Q H должен равняться проделанной работе, Вт, плюс тепло, которое должно рассеиваться в виде отработанного тепла . Q C в окружающую среду. Поэтому мы можем переписать формулу для теплового КПД как:

Поглощенное тепло происходит при сгорании топливно-воздушной смеси, когда возникает искра, примерно при постоянном объеме.Поскольку во время изохорного процесса система не выполняет никакой работы или над ней, первый закон термодинамики диктует ∆U = ∆Q. Следовательно, добавленное и отклоненное тепло определяется по формуле:

Q add = mc p (T 3 — T 2 )

Q out = mc v (T 4 — T 1 )

Подставив эти выражения для добавленного и отклоненного тепла в выражение для теплового КПД, получаем:

Это уравнение можно преобразовать в форму со степенью сжатия и степенью отсечки:

, где

  • η Дизель — максимальный тепловой КПД дизельного цикла
  • α — коэффициент отсечки V 3 / V 2 (т.е.е., соотношение объемов в конце и начале фазы сгорания)
  • CR — степень сжатия
  • κ = c p / c v = 1,4

Это очень полезный вывод, потому что желательно достичь высокой степени сжатия, чтобы извлечь больше механической энергии из данной массы топлива. Как было сказано в предыдущем разделе, тепловой КПД стандартного для воздуха цикла Отто также является функцией степени сжатия и κ.

Когда мы сравниваем их с формулами, можно видеть, что цикл Отто будет более эффективным для данной степени сжатия (CR), чем дизельный цикл. Но дизельные двигатели обычно более эффективны, поскольку могут работать при более высоких степенях сжатия.

В обычных двигателях Otto степень сжатия имеет свои пределы. Степень сжатия в бензиновом двигателе обычно не превышает 10: 1. Более высокие степени сжатия сделают бензиновые двигатели подверженными детонации, вызванной самовоспламенением несгоревшей смеси, если используется топливо с более низким октановым числом.Риск самовоспламенения топлива минимален, поскольку дизельные двигатели являются двигателями с воспламенением от сжатия и в начале такта сжатия в цилиндре нет топлива.

КПД двигателей на транспорте

  • В середине двадцатого века типичный паровоз имел тепловой КПД около 6% . Это означает, что на каждые 100 МДж сожженного угля было произведено 6 МДж механической энергии.
  • Типичный бензиновый автомобильный двигатель работает с тепловым КПД от 25% до 30% .Около 70-75% выбрасывается в виде отработанного тепла без преобразования в полезную работу, то есть работу, передаваемую на колеса.
  • Типичный автомобильный дизельный двигатель работает при примерно от 30% до 35% . В целом двигатели, использующие дизельный цикл, обычно более эффективны.
  • В 2014 году были введены новые правила для автомобилей Формулы 1 . Эти правила автоспорта подтолкнули команды к разработке высокоэффективных силовых агрегатов. По данным Mercedes, их силовой агрегат в настоящее время достигает более 45% и почти 50% теплового КПД, т.е.е. 45-50% потенциальной энергии топлива передается на колеса.
  • Дизельный двигатель имеет самый высокий тепловой КПД среди всех применяемых двигателей внутреннего сгорания. Низкооборотные дизельные двигатели (используемые на судах) могут иметь тепловой КПД, превышающий 50% . Самый большой дизельный двигатель в мире — 51,7%.

Среднее эффективное давление — MEP

MEP — полезная мера способности двигателя выполнять работу независимо от рабочего объема двигателя.

Параметр, используемый инженерами для описания характеристик поршневых двигателей, известен как среднее эффективное давление или MEP . MEP — полезный показатель способности двигателя выполнять работу независимо от рабочего объема двигателя. Есть несколько типов MEP. Эти MEP определяются методом измерения и расчета местоположения (например, BMEP или IMEP).

В общем, среднее эффективное давление — это постоянное теоретическое давление, которое могло бы создать такую ​​же сеть, как и в одном полном цикле, если бы оно действовало на поршень во время рабочего хода. MEP может быть определен как:

Например, чистое показало среднее эффективное давление , известное как IMEP n , равно среднему эффективному давлению, рассчитанному на основе давления в цилиндре (должно быть это измерение) за полный цикл двигателя.Обратите внимание, что это 720 ° для четырехтактного двигателя и 360 ° для двухтактного двигателя.

Некоторые примеры:

  • MEP атмосферного бензинового двигателя может находиться в диапазоне от 8 до 11 бар в области максимального крутящего момента.
  • MEP бензинового двигателя с турбонаддувом может составлять от 12 до 17 бар.
  • МЭП атмосферного дизельного двигателя может составлять от 7 до 9 бар.
  • MEP дизельного двигателя с турбонаддувом может находиться в диапазоне от 14 до 18 бар

Например, четырехтактный бензиновый двигатель, производящий 200 Н · м на 2 литрах рабочего объема, имеет MEP (4π) (200 Н · м) / (0.002 м³) = 1256000 Па = 12 бар. Как видно, MEP является полезной характеристикой двигателя . Для двух двигателей равного рабочего объема один с более высоким MEP на будет производить на большую сеть и, если двигатели работают с одинаковой скоростью, на большую мощность .

Дизельный цикл — проблема с решением

pV-диаграмма идеального дизельного цикла

Предположим, что дизельный цикл, который является одним из наиболее распространенных термодинамических циклов , которые можно найти в автомобильных двигателях .Одним из ключевых параметров таких двигателей является изменение объемов между верхней мертвой точкой (ВМТ) и нижней мертвой точкой (НМТ). Соотношение этих объемов ( V 1 / V 2 ) известно как степень сжатия . Кроме того, коэффициент отсечки составляет V 3 / V 2 , что представляет собой соотношение объемов в конце и начале фазы сгорания.

В этом примере предположим, что дизельный цикл со степенью сжатия CR = 20: 1 и коэффициентом отсечки α = 2.Воздух находится под давлением 100 кПа = 1 бар, 20 ° C (293 K), а объем камеры до такта сжатия составляет 500 см³.

  • Удельная теплоемкость при постоянном давлении воздуха при атмосферном давлении и температуре помещения: c p = 1,01 кДж / кгК.
  • Удельная теплоемкость при постоянном объеме воздуха при атмосферном давлении и температуре помещения: c v = 0,718 кДж / кгK.
  • κ = c p / c v = 1.4

Рассчитать:

  1. масса всасываемого воздуха
  2. температура T 2
  3. давление p температура 3
  4. количество тепла, добавляемого за счет сжигания топливовоздушной смеси
  5. тепловой КПД этого цикла
  6. MEP

Решение:

1) В начале расчетов мы должны определить количество газа в цилиндре перед тактом сжатия.Используя закон идеального газа, мы можем найти массу:

pV = mR specific T

где:

  • p — абсолютное давление газа
  • m is масса вещества
  • T — абсолютная температура
  • V — объем
  • R удельная — удельная газовая постоянная, равная универсальной газовой постоянной, деленной на газ или смесь молярная масса (M).Для сухого воздуха R удельный = 287,1 Дж. Кг -1 .K -1 .

Следовательно,

м = p 1 V 1 / R специфический T 1 = (100000 × 500 × 10 -6 ) / (287,1 × 293) = 5,95 × 10 -4 кг

2)

В этой задаче известны все объемы:

  • V 1 = V 4 = V max = 500 × 10 -6 м 3 (0.5l)
  • V 2 = V min = V max / CR = 25 × 10 -6 м 3

Обратите внимание, что (V max — V min ) x число цилиндров = общий рабочий объем двигателя

Поскольку процесс является адиабатическим, мы можем использовать следующее соотношение p, V, T для адиабатических процессов:

, таким образом,

T 2 = T 1 . CR κ — 1 = 293. 20 0,4 = 971 K

3)

Опять же, мы можем использовать закон идеального газа, чтобы найти давление в конце такта сжатия:

p 2 = mR специфический Т 2 / V 2 = 5.95 × 10 -4 x 287,1 x 971/25 × 10 -6 = 6635000 Па = 66,35 бар

4)

Поскольку процесс 2 → 3 происходит при постоянном давлении, уравнение состояния идеального газа дает

T 3 = (V 3 / V 2 ) x T 2 = 1942 K

Для расчета количества тепла, добавляемого при сгорании топливно-воздушной смеси, Q прибавляем , имеем чтобы использовать первый закон термодинамики для изобарного процесса, который гласит:

Q добавить = mc p (T 3 — T 2 ) = 5.95 × 10 -4 x 1010 x 971 = 583,5 Дж

5)

Тепловой КПД для этого дизельного цикла:

Как было выведено в предыдущем разделе, тепловой КПД дизельного цикла является функцией степень сжатия, степень отсечки и κ:

, где

  • η Дизель — максимальный тепловой КПД дизельного цикла
  • α — коэффициент отсечки V 3 / V 2 (т.е. соотношение объемов в конце и начале фазы сгорания)
  • CR — степень сжатия
  • κ = c p / c v = 1.4

Для этого примера:

η Дизель = 0,6467 = 64,7%

6)

MEP был определен как:

В этом уравнении рабочий объем равен V max — V мин . Чистая работа для одного цикла может быть рассчитана с использованием добавленного тепла и теплового КПД:

W нетто = Q добавить .

Автор: alexxlab

Добавить комментарий

Ваш адрес email не будет опубликован. Обязательные поля помечены *